Você está na página 1de 77

1

Which of the following cosmetic injectables does not illicit an inflammatory response?
1

Radiesse (hydroxyapatite)
2

Artecoll (polymethylmethacrylate)
3

Silicone
4

Zyplast (collagen)
5

Sculptra (poly L-lactic acid)
Q/Q(M)-475342 Report a Problem


Which of the following cosmetic injectables does not illicit an inflammatory response?
1

Radiesse (hydroxyapatite)
Radiesse is an injectable, biodegradable filler composed of calcium hydroxyapatite microspheres. It is a
normal constituent of bone and is therefore biocompatible. Current studies are examining the role of
hydroxyapatite in augmenting the craniofacial skeleton.
Q/Q(M)-475342 Report a Problem
While removing sutures from an excision performed on the lateral neck, your patient reports decreased
sensation of the helix and lobule of the ipsilateral ear. Which of the following nerves was likely injured
during surgery?
1

Auriculotemporal nerve
2

Lesser occipital nerve
3

Auricular branch of vagus nerve
4

Greater auricular nerve
5

Spinal accessory nerve
Q/Q(M)-474746 Report a Problem

While removing sutures from an excision performed on the lateral neck, your patient reports decreased
sensation of the helix and lobule of the ipsilateral ear. Which of the following nerves was likely injured
during surgery?
4

Greater auricular nerve
Sensory innervation of the ear is provided by the greater auricular nerve, the auriculotemopral nerve, the
lesser occipital nerve and the auricular branch of the vagus nerve. The greater auricular nerve is
composed of the C2 and C3 branches of the cervical plexus and lies in the posterior triangle of the neck.
It provides sensation to the helix, antihelix, antitragus, posterior auricle and the lobule. The spinal
accessory nerve is a motor nerve which innervates the trapezius muscle. This nerve is also subject to
injury in the lateral neck but would cause motor deficits rather than sensory deficits.
Q/Q(M)-474746 Report a Problem


Alopecia neoplastica is most commonly associated with which malignancy?
1

Lymphoma
2

Squamous cell carcinoma
3

Breast carcinoma
2

4

Colon carcinoma
5

Melanoma
Q/Q(M)-475362 Report a Problem


Alopecia neoplastica is most commonly associated with which malignancy?
3

Breast carcinoma
While various malignancies have a predilection for metastasis to the scalp where they can cause hair
loss, alopecia neoplastica is most commonly associated with metastatic breast carcinoma.
Q/Q(M)-475362 Report a Problem
What is the term used to describe the parallel nature of coherent light waves is?
1

monochromicity
2

coherence
3

collimation
4

energy
5

power
Q/Q(M)-482905 Report a Problem

What is the term used to describe the parallel nature of coherent light waves is?
3

Collimation
Collimation is the parallel nature of coherent light waves. Monochromaticity is the emission of a single
wavelength. Coherence is the term for light waves traveling in phase. Energy is the fundamental unit of
work. Power is the rate of energy delivery measured in watts.
Q/Q(M)-482905 Report a Problem


The role of imiquimod in the treatment of superficial basal cell carcinoma is based on the induction of
which cytokine?
1

Interleukin-2
2

Tumor growth factor-alpha
3

Interleukin-4
4

Interferon-gamma
5

Tumor necrosis factor-beta
Q/Q(M)-474754 Report a Problem

The role of imiquimod in the treatment of superficial basal cell carcinoma is based on the induction of
which cytokine?
4

Interferon-gamma
Imiquimod is an immunomodulator which induces the production of cytokines that are involved in cell-
mediated immunity. These cytokines include interferon-alpha, interferon-gamma, interleukin-1,
interleukin-10 and tumor necrosis factor-alpha. Induction of these cytokines results in antiviral and
3

antitumor activity in vivo.
Q/Q(M)-474754 Report a Problem

Which of the following cosmetic injectables can be seen on routine dental x-rays?
1

Botox
2

Zyplast
3

Radiesse
4

Cosmoderm
5

Restylane
Q/Q(M)-474714 Report a Problem


Which of the following cosmetic injectables can be seen on routine dental x-rays?
3

Radiesse
Radiesse is an injectable, biodegradable filler which is composed of calcium hydroxylapatite
microspheres. Calcium hydroxylapatite is a normal constituent of bone and thus can be seen on
radiographic imaging. Botox is botulinum toxin; Zyplast is bovine collagen; Cosmoderm is a non-animal
form of collagen; and Restylane is a non-animal form of hyaluronic acid.
Q/Q(M)-474714 Report a Problem



Which of the following suture materials is most likely to cause infection?
1

Silk
2

Vicryl (polyglactin)
3

Prolene (polypropylene)
4

Nylon
5

PDS (polydioxanone)
Q/Q(M)-474717 Report a Problem


Which of the following suture materials is most likely to cause infection?
1

Silk
Silk is a nonabsorbable braided suture which has been shown to aid in the production of infection. It is
believed that the interstices of braided or twisted suture materials enhance the potential for developing
infection by providing privileged sites which harbor bacteria.
Q/Q(M)-474717 Report a Problem
The most important function of a postsurgical wound dressing is to
1

Immobilize the wound
2

Prevent wound infection
3

Prevent scar formation
4

4

Reduce pain
5

Evert wound edges
Q/Q(M)-479604 Report a Problem


The most important function of a postsurgical wound dressing is to
1

Immobilize the wound
Postsurgical wound dressings are intended to aid in hemostasis, prevent infection and provide a semi-
occlusive environment. The most important postoperative function is to immobilize the wound and
surrounding tissue to allow the newly formed platelet plugs to remain intact. Winton GB, Salasche SJ.
Wound dressings for dermatologic surgery. J Am Acad Dermatol 1985; 13: 1026-44
Q/Q(M)-479604 Report a Problem

Which laser would not be a good choice for the treatment of a red cosmetic tattoo around the lips?
1

Carbon dioxide
2

Pulsed dye
3

Q-switched ruby
4

Alexandrite
5

Nd:YAG
Q/Q(M)-477355 Report a Problem

Which laser would not be a good choice for the treatment of a red cosmetic tattoo around the lips?
3

Q-switched ruby
Q-switched lasers can be effectively used to remove tattoos. However, immediate darkening of white,
flesh colored, and pink/red tattoos has been reported after treatment with Q-switched lasers. Both ferric
oxide and titanium dioxide (found in tattoo pigments) may be both be reduced after heating by the laser
and produce dramatic darkening.
Q/Q(M)-477355 Report a Problem


Botulinum toxin type A cleaves which protein in the presynaptic neuron?
1

Acetylcholine
2

Snap-25
3

Serotonin
4

Synaptobrevin (VAMP)
5

Syntaxin
Q/Q(M)-477184 Report a Problem


Botulinum toxin type A cleaves which protein in the presynaptic neuron?
2

Snap-25
Botulinum toxin type A is FDA approved for the treatment of glabellar wrinkles. Botulinum toxins act
5

by a three step process of binding, internalization by receptor medicated endocytosis and enzymatic
activation. It has specific light chain intracellular binding sites and different sites of action on different
SNARE (synaptosomal associated protein receptor [SNAP]) proteins. The SNARE proteins are
intimately involved in releasing acetylcholine at presynaptic terminals. Botulinum toxin A cleaves
SNAP-25 whereas B cleaves synaptobrevin.
Q/Q(M)-477184 Report a Problem
Which of the following cosmetic injectables binds water to create volume, has a low allergic reaction
profile, and lasts 6 to 12 months?
1

Botulinum toxin
2

Calcium hydroxylapatite
3

Human-derived collagen
4

Hyaluronic acid
5

Silicone
Q/Q(M)-474743 Report a Problem


Which of the following cosmetic injectables binds water to create volume, has a low allergic reaction
profile, and lasts 6 to 12 months?
4

Hyaluronic acid
Hyaluronic acid is a natural component of human connective tissue. As a cosmetic filler, hyaluronic acid
is an attractive alternative to currently available filler substances: its ability to bind water creates volume
and plumps the skin; its duration of action of approximately 6-12 months is appreciably longer than
conventional collagen; and since hyaluronic acid is chemically identical across all species,
hypersensitivity reactions are rare.
Q/Q(M)-474743 Report a Problem

Which of the following complications has been reported with infraorbital injections of Botox?
1

Nystagmus
2

Blindness
3

Festooning
4

Astigmatism
5

Photophobia
Q/Q(M)-475346 Report a Problem

Which of the following complications has been reported with infraorbital injections of Botox?
3

Festooning
Festooning of the lower eyelid is a recently reported complication of infraorbital Botox injections. The
affected patient had a prior lower lid blepharoplasty which likely weakened the orbicularis oculi muscle.
The author postulates that festooning resulted from further muscle weakness due to Botox treatment.
Q/Q(M)-475346 Report a Problem


6

A 78 year old man with an infiltrative basal cell carcinoma of the left nasal ala. what is the best
treatment option?
1

MOHS surgery
2

5-fluorouracil cream
3

Electrodessication and curettage
4

Standard excision
5

Imiquimod
Q/Q(M)-482826 Report a Problem


A 78 year old man with an infiltrative basal cell carcinoma of the left nasal ala. what is the best
treatment option?
1

MOHS surgery
Based on the histopathologic classification and location, this infiltrative basal cell carcinoma (BCC) is
best treated with MOHS surgery to preserve normal tissue and structure. 5-fluorouracil cream is used to
treat superficial BCC, squamous cell carcinoma in situ (SCCIS), and actinic keratoses.
Electrodessication and curettage is used to treat SCCIS and superficial BCC. Standard excision would
be possible if preservation of normal tissue and structure were not as crucial as in this area, the nasal ala.
Imiquimod is used to treat actinic keratoses and superficial basal cell carcinoma.
Q/Q(M)-482826 Report a Problem


Which is an advantage of a split-thickness skin graft over a full-thickness skin graft?
1

Better color match
2

Better texture match
3

Less contraction
4

Decreased infection risk
5

Improved survival
Q/Q(M)-482683 Report a Problem
Which is an advantage of a split-thickness skin graft over a full-thickness skin graft?
5

Improved survival
The major advantage of a split-thickness skin graft over a full-thickness graft is the increased
survivability. Full-thickness skin grafts have better color and texture match and less contraction. There is
no significant difference in infection risk.
Q/Q(M)-482683 Report a Problem


The nasolabial two stage flap uses depends on which artery for flap survival:
1

Supratrochlear artery
2

Superior labial artery
3

Angular artery
4

Lateral Nasal artery
7

5

Inferior labial artery
Q/Q(M)-477432 Report a Problem


The nasolabial two stage flap uses depends on which artery for flap survival:
3

Angular artery
Angular artery provides blood supply for the nasolabial transposition flap.
Q/Q(M)-477432 Report a Problem


Which of the following demonstrates the highest risk of metastasis?
1

SCC on the ear
2

SCC within a scar
3

SCC on the lip
4

SCC on the scalp
5

SCC on the nose
Q/Q(M)-474722 Report a Problem


Which of the following demonstrates the highest risk of metastasis?
2

SCC within a scar
Location of tumor is an important risk factor for metastasis of squamous cell carcinoma. Compared with
a 10% likelihood of metastasis for tumors located on the ear or the lip, an SCC developing in the scar,
however, has been estimated to metastasize at a rate as high as 30-40%. .
Q/Q(M)-474722 Report a Problem


How long after cutaneous infiltration of lidocaine with epinephrine is maximal vasoconstriction
achieved?
1

1 minute
2

7 minutes
3

15 minutes
4

30 minutes
5

1 hour
Q/Q(M)-474741 Report a Problem

How long after cutaneous infiltration of lidocaine with epinephrine is maximal vasoconstriction
achieved?
3

15 minutes
The use of epinephrine with local anesthesia has two main advantages. Firstly, the epinephrine is a
vasoconstrictor and assists in controlling bleeding during surgery. Secondly, and also a direct result of
its vasoconstrictive effects, epinephrine helps prolong the duration of the anesthetic agent 100% to
200% by delaying its absorption from the surgical site. Although the anesthetic properties of lidocaine
8

take effect within the first few minutes of infiltration, the vasoconstrictive effects of epinephrine require
approximately 15 minutes to fully develop.
Q/Q(M)-474741 Report a Problem
Which of the following determines the wavelength of a laser?
1

Fluence
2

Q switch
3

Medium
4

Spot size
5

Pulse duration
Q/Q(M)-477340 Report a Problem


Which of the following determines the wavelength of a laser?
3

Medium
The wavelength of a laser is determined by the medium.
Q/Q(M)-477340 Report a Problem


The type of repair depicted in the photograph is a:
1

Rhombic flap
2

Nasolabial flap
3

Rotation flap
4

Full-thickness skin graft
5

Island pedicle flap
Q/Q(M)-474977 Report a Problem


The type of repair depicted in the photograph is a:
9

1

Rhombic flap
The rhombic flap is a type of transposition flap. The classic rhombic flap is designed with two 60 degree
angles and two 120 degree angles. The point of maximum tension is at the closure of the donor site.
Q/Q(M)-474977 Report a Problem



All of the following are branches of the internal carotid artery except:
1

Ophthalmic artery
2

Angular artery
3

Supraorbital artery
4

Dorsal nasal artery
5

Supratrochlear artery
Q/Q(M)-477191 Report a Problem


All of the following are branches of the internal carotid artery except:
2

Angular artery
Branches of the internal carotid artery include the ophthalmic artery, supraorbital artery, dorsal nasal
artery, supratrochlear artery, palpebral artery, and lacrimal artery. Branches of the external carotid artery
include the superficial temporal artery, facial artery (inferior labial, superior labial, angular branches),
maxillary artery, posterior auricular, and occipital artery. The external and internal carotid arteries
anastamose in the periorbital region.
Q/Q(M)-477191 Report a Problem

Which ocular structure is at most risk of injury from an erbium: YAG laser?
1

Lens
2

Retina
3

Cornea
4

Vitreous humor
5

Iris
Q/Q(M)-477205 Report a Problem


Which ocular structure is at most risk of injury from an erbium: YAG laser?
3

Cornea
Injury from lasers may occur via direct or indirect ocular exposure. Damage is generally wavelength
specific. Laser that target hemoglobin or pigment may cause damage to the retinal pigment or
vasculature while lasers that target water as a chromophore (carbon dioxide and erbium) can damage the
cornea.
Q/Q(M)-477205 Report a Problem


10


Which nerve provides the sensory innervation to the tragus?
1

Auricular branch of vagus
2

Facial
3

Glossopharyngeal
4

Auriculotemporal
5

Greater auricular
Q/Q(M)-477513 Report a Problem


Which nerve provides the sensory innervation to the tragus?
4

Auriculotemporal
The auriculotemporal nerve which is a branch of the mandibular branch of the trigeminal nerve
innervates the tragus. The sensory innervation to the auricle is provided by the greater auricular nerve,
and to a lesser extent the lesser occipital nerve. The external auditory canal, concha, and posauricular
sulcus supplied variably by 3 nerves - the auricular branch of the vagus (CNX), the facial nerve (CN7),
and the glossopharyngeal (CNIX).
Q/Q(M)-477513 Report a Problem

A 60-year-old man receives cephalexin prophylaxis 60 minutes prior to his Mohs surgery for a BCC on
the nose. Which condition of his most likely necessitated prophylaxis?
1

History of CABG
2

Secundum atrial septal defect
3

Mitral valve prolapse with regurgitation
4

History of rheumatic fever without valve dysfunction
5

Pacemaker
Q/Q(M)-482389 Report a Problem



A 60-year-old man receives cephalexin prophylaxis 60 minutes prior to his Mohs surgery for a BCC on
the nose. Which condition of his most likely necessitated prophylaxis?
3

Mitral valve prolapse with regurgitation
Prior to Mohs patients who are at high risk should receive prophylaxis int he form of cephalexin (or
clindamycin or azithromycin if penicillin-allergic) 30-60 minutes prior to the procedures for non-oral
skin. High risk patients include those with prosthetic valves, a history of bacterial endocarditis, mitral
valve prolapse with regurgitation, mitral valve prolapse without regurgitation in MEN > 45 years, any
valve dysfuntion, cardiac malformation, hypertrophic cardiomyoopathy, CNS shunts, and more. Some
low-risk examples (no prophylaxis) include s/p CABG, pacemaker, murmur, or mitral valve prolapse
without regurgitation.
Q/Q(M)-482389 Report a Problem
What part of the eye may be damaged by exposure to irradiation from the carbon dioxide laser?
1

Retina
11

2

Lens
3

Sclera
4

Cornea
5

Iris
Q/Q(M)-476563 Report a Problem


What part of the eye may be damaged by exposure to irradiation from the carbon dioxide laser?
4

Cornea
The carbon dioxide laser operates at a wavelength of 10,600 nm and targets water as a chromophore.
Because of the high water content of the cornea, it may be damaged by exposure to irradiation from the
carbon dioxide laser. Exposure to the erbium may cause corneal damage as well.
Q/Q(M)-476563 Report a Problem


Cosmetic units are defined by each of the following anatomic boundaries except:
1

Nasal tip
2

Glabellar region
3

Nasofacial sulcus
4

Philtral ridge
5

Nasolabial folds
Q/Q(M)-474755 Report a Problem

Cosmetic units are defined by each of the following anatomic boundaries except:
1

Nasal tip
The term cosmetic unit is used to define regions of the face by specific characteristics such as color,
texture, presence of hair, and prominence of sebaceous glands. In dermatologic surgery, it is preferable
to conceal incisions within the boundary lines of cosmetic units. In addition, moving skin from one
cosmetic unit to another should be avoided when possible to minimize the appearance of apposed skin of
dissimilar quality.
Q/Q(M)-474755 Report a Problem

Which topical antiseptic works via denaturation of proteins?
1

Alcohols
2

Chlorhexidine
3

Iodine
4

Hexachlorophene
5

Triclosan
Q/Q(M)-482388 Report a Problem


12


Which topical antiseptic works via denaturation of proteins?
1

Alcohols
Alcohols exert their antimicrobials effect via denaturation of proteins ie DNA, RNA, lipids, etc.
Chlorhexidine, hexachlorophene, and triclosan disrupt the microbial cell membrane. Iodine and
iodophores work via oxidation/substitution for elemental form of iodine, thereby precipitating proteins.
Q/Q(M)-482388 Report a Problem



The photograph demonstrates which of the following?
1

Split-thickness skin graft
2

Mohs layer
3

Full-thickness skin graft
4

Dog ear
5

Composite graft
Q/Q(M)-474972 Report a Problem


The photograph demonstrates which of the following?
3

Full-thickness skin graft
A full-thickness skin graft (FTSG) is composed of the entire epidermis and dermis. Subcutaneous tissue
must be removed since the fat may compromise the viability of the graft.
Q/Q(M)-474972 Report a Problem


When can a patient who undergoes follicular-unit hair transplantation expect hair growth of the grafted
follicles to begin?
1

7 to 10 days
2

1 month
3

2 to 3 months
4

6 months
5

9 months
Q/Q(M)-474738 Report a Problem
13




When can a patient who undergoes follicular-unit hair transplantation expect hair growth of the grafted
follicles to begin?
3

2 to 3 months
The use of follicular-unit grafts, which contain one to four hair follicles, represents the advancement in
both surgical technique and aesthetic outcome in the field of hair transplantation. The grafted hair
follicles typically begin to grow within 8 to 10 weeks of implantation and are expected to survive for the
individuals lifetime.
Q/Q(M)-474738 Report a Problem


Which of the following is a complication of Thermage treatment?
1

Keloid formation
2

Lipoatrophy
3

Granuloma formation
4

Hyperpigmentation
5

Herpes simplex infection
Q/Q(M)-475345 Report a Problem



Which of the following is a complication of Thermage treatment?
2

Lipoatrophy
Thermage (ThermaCool) is a nonablative radiofrequency-based system which uses volumetric heating to
induce tightening of the skin and dermal remodeling. Lipoatrophy is a late-occurring potential
complication of this technology.
Q/Q(M)-475345 Report a Problem


Which of the following local anesthetic agents should not be used in children?
1

Bupivicaine
2

Etidocaine
3

Mepivicaine
4

Prilocaine
5

Benzocaine
Q/Q(M)-477394 Report a Problem

Which of the following local anesthetic agents should not be used in children?
4

Prilocaine
Prilocaine should bot be used in children given the risk of methemoglobinemia.Methemoglobulinemia in
children can occur from exposure to oxidizing substances such as aniline dyes, prilocaine or pyrimidine.
14

In methemoglobinemia, the iron in hemoglobin is oxidized from the ferrous state (Fe 2+) to ferric
(Fe3+) resulting in the inability to transport oxygen and carbon dioxide. Clinically, this condition results
in cyanosis.
Q/Q(M)-477394 Report a Problem


Which of the following cosmetic injectables has the longest duration of action?
1

Botox
2

Sculptra (poly L-lactic acid)
3

Restylane (hyaluronic acid)
4

Cosmoderm (collagen)
5

Myobloc
Q/Q(M)-475352 Report a Problem

Which of the following cosmetic injectables has the longest duration of action?
2

Sculptra (poly L-lactic acid)
Sculptra (called New-Fill outside of the US) is a biodegradable filler composed of poly-L-lactic acid, the
same material used in absorbable sutures. Preliminary studies have demonstrated longevity of the filler
at two years post-treatment. Valantin MA et al. Polylactic acid implants (New-Fill) to correct facial
lipoatrophy in HIV-infected patients: results of the open-label study VEGA. AIDS. 2003;17:2471-2477.
Q/Q(M)-475352 Report a Problem



A Z- plasty is performed to:
1

Reorient a scar
2

Shorten the length of an excision
3

Close a wound on a convex surface
4

Obtain wound eversion
5

Make use of skin from an area of laxity
Q/Q(M)-477326 Report a Problem

A Z- plasty is performed to:
1

Reorient a scar
A Z-plasty is a type of transposition flap that may be used to reorient a scar, especially when the scar
crosses skin tension lines. It is also used to change the scar length or vector and ease scar contractures.
Q/Q(M)-477326 Report a Problem

Paradoxical darkening has been associated with Q-switched ruby, Q-switched alexandrite, Q-switched
Nd:Yag treatment of which tattoo color?
1

Green
15

2

Red
3

Yellow
4

Black
5

Blue
Q/Q(M)-476762 Report a Problem



Paradoxical darkening has been associated with Q-switched ruby, Q-switched alexandrite, Q-switched
Nd:Yag treatment of which tattoo color?
2

Red
Paradoxic darkening of flesh-tone, red, and white tattoo inks with QSRL, Q-switched Nd:YAG, and Q-
switched alexandrite lasers has been reported. The tattoo pigments most associated with this
phenomenon are iron oxide and titanium dioxide. Ferric sulfide is converted to ferrous sulfate by the
laser.
Q/Q(M)-476762 Report a Problem


A patient who had liposuction 5 weeks ago presents with multiple firm nodules at the cannula insertion
sites. You suspect the diagnosis is:
1

Foreign body granuloma
2

Herpes simplex infection
3

Organized hematoma
4

Mycobacterial infection
5

Cold panniculitis
Q/Q(M)-475369 Report a Problem

A patient who had liposuction 5 weeks ago presents with multiple firm nodules at the cannula insertion
sites. You suspect the diagnosis is:
4

Mycobacterial infection
Atypical mycobacterial infections are occurring with increasing frequency after cosmetic surgery
procedures. These infections typically occur 4-14 weeks after a procedure as a late-occurring
complication. Firm nodules at the treatment site or dehiscence of a previously healed wound may be
presenting signs of atypical mycobacterial infection.
Q/Q(M)-475369 Report a Problem



Which of the following lasers uses a solid as its medium?
1

Argon laser
2

pulded dye laser
3

Alexandrite
4

Krypton
16

5

carbon dioxide
Q/Q(M)-482904 Report a Problem


Which of the following lasers uses a solid as its medium?
3

Alexandrite
Solid media lasers include alexandrite, KTP, ruby, Nd Yag, diode. Liquid media is found in the pulsed
dye laser. Gas media is used in argon, carbon dioxide, krypton, and cooper vapor laser.
Q/Q(M)-482904 Report a Problem


Which post-operative complication is shown in the photograph?
1

Trap door deformity
2

Hypertrophic scar
3

Ectropion
4

Temporal nerve paralysis
5

Eclabium
Q/Q(M)-474978 Report a Problem



Which post-operative complication is shown in the photograph?
3

Ectropion
Ectropion occurs when the eyelid margin turns outward away from the eye. In lower eyelid
reconstruction, tension should be oriented parallel to the lower eyelid to prevent the development of
ectropion.
Q/Q(M)-474978 Report a Problem



Which of the following chemical peels does not need to be neutralized?
1

Salicylic acid
2

TCA 40%
3

Glycolic acid
4

Lactic acid
5

Phenol
Q/Q(M)-482456 Report a Problem
17



Which of the following chemical peels does not need to be neutralized?
1

Salicylic acid
Salicylic acid is a very superficial chemical peel that is used for acne, milia, keratolysis. This peel
localizes to the pores given its lipophilic nature. It is a self-neutralizing peel.
Q/Q(M)-482456 Report a Problem


The cutaneous lip and chin are divided into cosmetic units by which anatomic boundary?
1

Vermillion border
2

Nasolabial fold
3

Marionette lines
4

Mental crease
5

Philtral ridge
Q/Q(M)-475488 Report a Problem


The cutaneous lip and chin are divided into cosmetic units by which anatomic boundary?
4

Mental crease
The term cosmetic unit is used to define regions of the face by specific characteristics such as color,
texture, presence of hair, and prominence of sebaceous glands. In dermatologic surgery, it is preferable
to conceal incisions within the boundary lines of cosmetic units. The mental crease, as demonstrated in
the photograph, divides the cutaneous lip and chin.
Q/Q(M)-475488 Report a Problem


Damage to the zygomatic branch of the facial nerve leads to which of the following:
1

Facial asymmetry
2

Inability to pucker lips
3

Eyelid ectropion and inability to close eyelid
4

Unilateral eyelid ptosis
5

Winged scapula
Q/Q(M)-477438 Report a Problem

Damage to the zygomatic branch of the facial nerve leads to which of the following:
18

3

Eyelid ectropion and inability to close eyelid
Damage to the zygomatic nerve causes eyelid ectropion and inability to close eyelid.
Q/Q(M)-477438 Report a Problem

The most appropriate suture for closing a wound on the buccal mucosa is:
1

Prolene
2

Silk
3

Monocryl
4

Braided nylon
5

PDS
Q/Q(M)-477162 Report a Problem

The most appropriate suture for closing a wound on the buccal mucosa is:
2

Silk
The most appropriate suture for closing a wound on the buccal mucosa is silk. Other sutures may be
used as well, but classically, silk is used because of its soft nature.
Q/Q(M)-477162 Report a Problem

Which of the following lasers has the greatest depth of penetration in the skin?
1

Pulsed dye laser (585 nm)
2

Diode (800 nm)
3

Nd: YAG (1064 nm)
4

Erbium: YAG (2940 nm)
5

CO2 (10,600 nm)
Q/Q(M)-474742 Report a Problem



Which of the following lasers has the greatest depth of penetration in the skin?
3

Nd: YAG (1064 nm)
The Nd: YAG laser emits energy at 1064nm near the infrared range and penetrates the skin to the level
of the deep dermal blood vessels. The target chromophore for this laser is melanin, thus its primary use
is in treating lesions such as nevus of Ota and removing black tattoo pigment. Although both the
erbium:YAG and CO2 lasers have longer wavelengths than the Nd: YAG, they are ablative lasers which
only penetrate to the level of the stratum corneum and superficial epidermis, respectively.
Q/Q(M)-474742 Report a Problem

The most common adverse reaction seen with betadine is:
1

Ototoxity
2

Teratogenicity
19

3

Allergic contact dermatitis
4

Seizures
5

Corneal damage
Q/Q(M)-477418 Report a Problem
The most common adverse reaction seen with betadine is:
3

Allergic contact dermatitis
Most common side effect seen with betadine is allergic contact dermatitis, secondary to the iodine
component.
Q/Q(M)-477418 Report a Problem


Which needle shape is most commonly used in cutaneous surgery?
1

1/4 circle
2

3/8 circle
3

1/2 circle
4

5/8 circle
5

compound curve
Q/Q(M)-475356 Report a Problem


Which needle shape is most commonly used in cutaneous surgery?
2

3/8 circle
Needles are either straight or curved. Curved needles have their curvature described either as a fraction
of a circle or a compound curve. The greater the fraction of a circle, the more pronation and supination
of the wrist required by the surgeon to place the needle. The 3/8 circle needle is easy to use in large,
superficial areas and is the most commonly used needle for cutaneous surgery.
Q/Q(M)-475356 Report a Problem


All of the following statements are true regarding imiquimod (Aldara) except:
1

It is FDA-approved for the treatment of actinic keratoses
2

It is FDA-approved for the treatment of a 1.2 cm superficial basal cell on the scalp
3

It is FDA-approved for the treatment of a 1.9 cm superficial basal cell on the chest
4

It is FDA-approved for the treatment of a 1.5 cm superficial basal cell on the leg
5

It is FDA-approved for the treatment of a 2.0 cm superficial basal cell on the back
Q/Q(M)-475370 Report a Problem


All of the following statements are true regarding imiquimod (Aldara) except:
2

It is FDA-approved for the treatment of a 1.2 cm superficial basal cell on the scalp
Imiquimod (Aldara) is FDA-approved for the treatment of actinic keratoses and the treatment of
superficial basal carcinoma. With respect to basal cell carcinoma, treatment is indicated for primary
20

tumors that are 2.0 cm or less, and that are located on areas of the body excluding the face, scalp and
anogenital region.
Q/Q(M)-475370 Report a Problem


Which anatomic structure is likely to be severed during this repair?
1

Mental nerve
2

Angular artery
3

Buccal nerve
4

Labial artery
5

Marginal mandibular nerve
Q/Q(M)-474975 Report a Problem

Which anatomic structure is likely to be severed during this repair?
4

Labial artery
The photograph demonstrates a wedge resection which is used to repair defects of the lower lip. During
this repair, the labial artery will be severed since it courses superficially between the mucosa and the
underlying muscle.
Q/Q(M)-474975 Report a Problem

Treatment of acne scarring would be most effective with which modality?
1

Pulsed dye laser
2

Intense pulsed light
3

Photodynamic therapy
4

Radiofrequency
5

Nd:YAG laser
Q/Q(M)-475373 Report a Problem

Treatment of acne scarring would be most effective with which modality?
5

Nd:YAG laser
Treatment with the nonablative 1064-nm Q-switched Nd:YAG provides significant improvement in skin
topography in patients with atrophic acne scars. The further improvements that are seen at 6-month
follow-up suggest that dermal remodeling is a process that continues long after treatment.
Q/Q(M)-475373 Report a Problem
21


The tensile strength of a wound 6 months after surgery compared to intact skin is:
1

15%
2

30%
3

50%
4

70%
5

100%
Q/Q(M)-474725 Report a Problem

The tensile strength of a wound 6 months after surgery compared to intact skin is:
4

70%
The tensile strength of a wound approaches 70% of normal skin strength at 8 weeks postoperatively.
Wound healing is a process which takes many months, yet the tensile strength of a wound never exceeds
80% of the tensile strength of intact skin. It is therefore critical to use long-lasting subcuticular sutures to
minimize the tension on a healing wound and prevent scar widening.
Q/Q(M)-474725 Report a Problem

Healing by secondary intention will offer the most favorable cosmetic result at which location?
1

Nasal tip
2

Malar cheek
3

Chin
4

Forehead
5

Medial canthus
Q/Q(M)-474729 Report a Problem

Healing by secondary intention will offer the most favorable cosmetic result at which location?
5

Medial canthus
Indications for healing of full-thickness skin wounds by secondary intention include infected wounds,
patients who refuse reconstructive surgery or who are poor surgical risks for reconstructive surgery, and
resection of tumors with high rates of recurrence. In addition, secondary intention, or granulation, can be
utilized when the cosmetic result is anticipated to be superior or equal to reconstruction. Disadvantages
of healing by granulation include prolonged wound healing time and somewhat unpredictable cosmetic
results. Anatomic sites lending themselves best to second intention healing, with excellent results, are
the concave areas on the face. These include the medial canthus, the nose-cheek junction, the auricular
concha, the preauricular cheek, and the retroauricular scalp. The nasal tip, malar cheek, chin and
forehead are all convex surfaces on the face.
Q/Q(M)-474729 Report a Problem


A 40 year-old woman from Southern California has wrinkles at rest on her forehead, scattered
telangiectasia on her nose and a few seborrheic keratoses on her chest. What category in Glogau's
photoaging classification scale does this patient represent?
22

1

Type 1
2

Type 2
3

Type 3
4

Type 4
5

Type 5
Q/Q(M)-474737 Report a Problem


A 40 year-old woman from Southern California has wrinkles at rest on her forehead, scattered
telangiectasia on her nose and a few seborrheic keratoses on her chest. What category in Glogau's
photoaging classification scale does this patient represent?
3

Type 3
Glogaus photoaging scale is a classification system that employs clinical markers of photodamage to
determine an individuals level of photoaging. The scale ranges from Type 1 through Type 4, with
Type 4 demonstrating the most extensive photodamage. Type 3 on the Glogau scale is classified as
advanced photoaging and describes a patient who is typically 50 years of age or older, has wrinkles at
rest, telangiectasias, obvious dyschromias and visible keratoses.
Q/Q(M)-474737 Report a Problem


Which one of the following best estimates the percent strength of a wound one month after surgery with
a primary closure?
1

30%
2

50%
3

70%
4

80%
5

90%
Q/Q(M)-477492 Report a Problem

Which one of the following best estimates the percent strength of a wound one month after surgery with
a primary closure?
2

50%
One month after surgery with a primary closure a wound is approximately at 50% strength. Two weeks
after surgery it is at 5% strength. Three weeks after surgery it is at 20%. It never reaches 100% of its
original strength, its maximum strength is 90%.
Q/Q(M)-477492 Report a Problem
Jessners solution contains all of the following except:
1

Salicylic acid
2

Resorcinol
3

Glycolic acid
4

Ethanol
23

5

Lactic acid
Q/Q(M)-477361 Report a Problem


Jessners solution contains all of the following except:
3

Glycolic acid
Jessners solution is a combination of resorcinol, salicylic acid, and lactic acid in ethanol that is used as
a superficial peeling agent. The advantage of Jessners solution is that timing is unnecessary and
neutralization is not performed. The lactic acid is an alpha hydroxy acid.
Q/Q(M)-477361 Report a Problem

Which of the forms of electrosurgery can work in a wet field?
1

Electrocautery
2

Electrodessication
3

Electrofulguration
4

Electrosection
5

Electrocoagulation
Q/Q(M)-477408 Report a Problem


Which of the forms of electrosurgery can work in a wet field?
1

Electrocautery
Electrocautery is the only one to work in a wet field as it has no current and works via heat.
Q/Q(M)-477408 Report a Problem


An M-plasty is performed to:
1

Decrease tension on a wound
2

Obtain wound eversion
3

Shorten the length of an excision
4

Reorient a scar
5

Make use of excess tissue
Q/Q(M)-474723 Report a Problem


An M-plasty is performed to:
3

Shorten the length of an excision
An M-plasty is a variation of the fusiform excision where either one or both ends of the ellipse are
modified. The main advantages to performing an M-plasty are to shorten the length of an excision and to
correct dog-ears.
Q/Q(M)-474723 Report a Problem
24


All of the following thrombotic complications have been reported after discontinuing aspirin therapy
prior to surgery except:
1

Stroke
2

Myocardial infarction
3

Pulmonary embolism
4

Transient ischemic attack
5

Cerebral embolism
Q/Q(M)-477214 Report a Problem

All of the following thrombotic complications have been reported after discontinuing aspirin therapy
prior to surgery except:
3

Pulmonary embolism
Kovich et al. reported thrombotic complications in patients who had stopped either coumadin or aspirin
peri-operatively. Thrombotic complications in patients who stopped warfarin included stroke, TIA,
myocardial infarction, cerebral embolism, death, DVT, pulmonary embolus, and blindness. Thrombotic
complications in patients who stopped aspirin included stroke, TIA, myocardial infarction, cerebral
embolism and death. No DVT or pulmonary embolus was reported.
Q/Q(M)-477214 Report a Problem
Blue nodules in the skin are a potential complication of which cosmetic injectable?
1

Radiance (hydroxyapatite)
2

Silicone
3

Restylane (hyaluronic acid)
4

Cosmoderm (collagen)
5

Sculptra (poly L-lactic acid)
Q/Q(M)-475351 Report a Problem
Blue nodules in the skin are a potential complication of which cosmetic injectable?
3

Restylane (hyaluronic acid)
Although hyaluronic acid is believed to be inert in humans, rare reports exist of granulomatous foreign
body reactions to this material. Skin biopsy of a Restylane nodule demonstrates multinucleated giant
cells surrounding a blue amorphous material.
Q/Q(M)-475351 Report a Problem


The use of topical vitamin K has been shown to:
1

Decrease the appearance of infraorbital pigmentation
2

Minimize the appearance of telangiectasia
3

Reduce the severity of laser-induced purpura
4

Increase collagen production
25

5

Induce keratinocyte differentiation
Q/Q(M)-474726 Report a Problem


The use of topical vitamin K has been shown to:
3

Reduce the severity of laser-induced purpura
Treatment of benign vascular lesions with the pulsed dye laser often produces significant postoperative
purpura. Topical vitamin K has been shown to decrease the severity of laser-induced purpura although
its mechanism of action has yet to be determined. No other cosmetic effects of topical vitamin K have
been proven to be statistically significant.
Q/Q(M)-474726 Report a Problem

Which of the following injectable local anesthetics is the longest lasting?
1

Lidocaine
2

Bupivacaine
3

Meppivacaine
4

Procaine
5

Diphenhydramine
Q/Q(M)-478614 Report a Problem

Which of the following injectable local anesthetics is the longest lasting?
2

Bupivacaine
Bupivacaine is the longest lasting injectable local anesthetic-120-240 minutes without epinephrine, 180-
420 minutes with epinephrine. Lidocaine lasts 30-60 minutes without epinephrine, 120-360 with
epinephrine. Mepivacaine last 45-90 minutes without epinephrine, 120-360 with epinephrine. Procaine
lasts 15-60 minutes. Diphenhydramine lasts 30 minutes.
Q/Q(M)-478614 Report a Problem

Which of the following parameters determines the wavelength of a laser?
1

Medium
2

Fluence
3

Spot size
4

Pulse duration
5

Lens length
Q/Q(M)-474724 Report a Problem
Which of the following parameters determines the wavelength of a laser?
1

Medium
Laser light is monochromatic light that is emitted at a single wavelength. The wavelength of the laser is
determined by the medium in the optical cavity of the laser through which the light passes. The medium
may be solid, liquid or gas.
26

Q/Q(M)-474724 Report a Problem


The use of dermabrasion to improve the cosmesis of a scar is best performed how long after the initial
surgery?
1

1 week
2

3 weeks
3

6 weeks
4

6 months
5

1 year
Q/Q(M)-476928 Report a Problem
The use of dermabrasion to improve the cosmesis of a scar is best performed how long after the initial
surgery?
3

6 weeks
Dermabrasion is the process of surgically planing or abrading the epidermis and dermis and is usually
carried out with a rapidly rotating wire brush or diamond fraise. Following dermabrasion,
reepithelialization from adnexal structures occurs with remodeling and replacement of collagen bundles
in the papillary and reticular dermis. This can result in improvement of surgical or acne scars. When
used in the post-operative period for surgical scars, dermabrasion is usually carried out 6 to 8 weeks
following the procedure. Regional dermabrasions are routinely carried out under local anesthesia. Post-
procedure complications include scarring, pigmentary alterations, persistent erythema, and infection.
Contraindications for dermabrasion include recent use of isotretinoin and recent facelift, browlift or
other procedure involving extensive undermining. Reactivation of herpes labialis can be prevented in the
lip area by administering antiviral prophylaxis.
Q/Q(M)-476928 Report a Problem

Which of the following topical antibacterial agents demonstrates activity against Pseudomonas species?
1

Mupirocin
2

Erythromycin
3

Polymyxin
4

Bacitracin
5

Clindamycin
Q/Q(M)-474749 Report a Problem


Which of the following topical antibacterial agents demonstrates activity against Pseudomonas species?
3

Polymyxin
Polymyxins are peptides produced by the organism Bacillus polymyxa. They are bactericidal against
Pseudomonas aeruginosa, Proteus mirabilis, Enterobacter, Klebsiella and Escherichia coli. Since they
provide little coverage against gram-positive organisms, polymyxins are often combined with other
antibacterial agents to increase their spectrum of activity.
Q/Q(M)-474749 Report a Problem


27

Treatment with which of the following lasers has been effective in psoriasis?
1

Carbon dioxide laser
2

Excimer laser
3

Ruby Laser
4

Pulsed dye laser
5

Alexandrite laser
Q/Q(M)-477359 Report a Problem

Treatment with which of the following lasers has been effective in psoriasis?
2

Excimer laser
The excimer laser is a 308 nm wavelength laser that has been used to treat psoriasis. The excimer laser
has some advantages over ultraviolet therapy. By treating only involved skin, higher doses can be used
and clearance may occur with fewer treatments.
Q/Q(M)-477359 Report a Problem



The repair demonstrated in the photograph is a:
1

Split-thickness skin graft
2

Interpolation flap
3

Full-thickness skin graft
4

Rotation flap
5

Transposition flap
Q/Q(M)-474974 Report a Problem


28

The repair demonstrated in the photograph is a:
2

Interpolation flap
The postauricular flap is useful for repairing extensive defects of the helical rim, particularly when
cartilage is involved. It is an example of an interpolation flap.
Q/Q(M)-474974 Report a Problem


Which of the following criteria carries the worst prognosis for a patient with a squamous cell
carcinoma?
1

Size of tumor > 1 cm
2

Depth of invasion > 4 mm
3

Perineural invasion
4

Anatomic location
5

Immunosuppression
Q/Q(M)-474730 Report a Problem


Which of the following criteria carries the worst prognosis for a patient with a squamous cell
carcinoma?
2

Depth of invasion > 4 mm
Many factors contribute to the aggressive behavior of cutaneous squamous cell carcinoma. For example,
tumors in immunosuppressed patients and tumors with the histologic subtypes desmoplastic and
acantholytic are often biologically aggressive squamous cell carcinomas. Compared with other risk
factors, depth of invasion >4 mm and size > 2cm demonstrate the greatest risk for metastasis.
Q/Q(M)-474730 Report a Problem

A surgeon plans to repair a defect on the lower lip with a wedge resection. All of the following
statements are true regarding wedge resection of the lower lip except?
1

a V-shaped excision is often used
2

the vermillion border is marked prior to excision
3

the repair involves a two layered closure of skin and mucosa
4

up to 1/3 of the lip can be excised and closed primarily
5

the mental crease should not be crossed
Q/Q(M)-475359 Report a Problem

A surgeon plans to repair a defect on the lower lip with a wedge resection. All of the following
statements are true regarding wedge resection of the lower lip except?
3

the repair involves a two layered closure of skin and mucosa
The lower lip has such great elasticity that full-thickness lesions that involve up to 1/3 of the lip can be
excised and repaired primarily with excellent cosmetic and functional results. This type of repair
requires a three layered closure of mucosa, skin and muscle. Larrabee W. In: Principles of Facial
Reconstruction. Lippincott-Raven: Philadelphia, 1995; 182-183.
29

Q/Q(M)-475359 Report a Problem


Sensory innervation of the glabella and mid-forehead is provided by which nerve?
1

infratrochlear nerve
2

supraorbital nerve
3

supratrochlear nerve
4

temporal branch of the facial nerve
5

zygomatic branch of the facial nerve
Q/Q(M)-475866 Report a Problem

Sensory innervation of the glabella and mid-forehead is provided by which nerve?
3

supratrochlear nerve
The supratrochlear, supraorbital and infratrochlear nerves are branches of the ophthalmic division of the
trigeminal nerve, supplying sensory innervation to the face. The supratrochlear nerve exits the orbit
approximately one centimete lateral to the midline and traverses upward along the medial supraorbital
rim along the supratrochlear notch. It provides sensory innervation to the medial upper eyelid and
glabella, the medial anterior forehead, and the scalp. The supraorbital nerve supplies the lateral forehead
and scalp as well as the upper eyelid. The infratrochlear nerve provides sensory innervation to the root
of the nose and to portions of the medial canthus. The facial nerve provides motor innervation to
muscles of the face. The temporal branch of the facial nerve supplies motor innervation to the upper
orbicularis oculi and the frontalis muscle, as well as the anterior and superior auricular muscles. The
zygomatic branch of the facial nerve provides innervation to the inferior portion of the orbicularis oculi,
zygomaticus minor and zygomaticus major muscles, as well as levator labii superioris. (Leffell's Manual
of Skin Surgery. 1997 p44.) (Surgical Anatomy of the Skin, Salasche et al, p 113-115.)
Q/Q(M)-475866 Report a Problem

Which of the following has been implicated in the promotion of skin aging?
1

alpha-tocopherol
2

Homocysteine
3

Proanthocyanidin
4

beta carotene
5

alpha-lipoic acid
Q/Q(M)-482470 Report a Problem


Which of the following has been implicated in the promotion of skin aging?
2

Homocysteine
Homocysteine is a sulfhydryl-containing amino acid produced during the conversion of methionine to
cysteine. Homocysteine can promote defects in fibrillin-1 and stimulate matrix metalloproteinase
production, leading to collagen and elastic fiber degradation. Homocystinuria, caused by a deficiency in
cystathionine beta-synthase, presents with osteoporosis, thin skin, and striae.
Q/Q(M)-482470 Report a Problem
30



All of the following are true statements regarding the immunohistochemical stain Melan-A except:
1

Antigen present in 80-100% of melanomas
2

Recognized by CD8+ T cells
3

Sensitive marker for melanocytic nevi
4

Effective in frozen and paraffin-embedded tissue
5

Reliably stains desmoplastic melanoma
Q/Q(M)-474748 Report a Problem


All of the following are true statements regarding the immunohistochemical stain Melan-A except:
5

Reliably stains desmoplastic melanoma
E Melan-A is a 22-kDa cytoplasmic melanosome-associated glycoprotein also known as MART-1
(melanoma antigen recognized by T cells). It is a sensitive marker for both melanocytic nevi and
melanoma and may be used on frozen tissue sections as well as paraffin-embedded tissue. Although the
antigen is present in > 80% of melanomas, it does not reliably stain desmoplastic or spindle cell
melanomas.
Q/Q(M)-474748 Report a Problem

Which one of the following lasers would be effective in the treatment of rhinophyma?
1

Pulsed dye laser
2

Ruby laser
3

Alexandrite laser
4

KTP laser
5

Carbon dioxide laser
Q/Q(M)-477219 Report a Problem


Which one of the following lasers would be effective in the treatment of rhinophyma?
5

Carbon dioxide laser
The carbon dioxide laser is a 10,600 nm laser that can be used to treat rhinophyma. The advantage of the
carbon dioxide laser over steel or dermabrasion is that the laser is relatively bloodless. The erbium YAG
can also be used to treat rhinophyma.
Q/Q(M)-477219 Report a Problem



Which statement is true regarding relaxed skin tension lines (RSTLs)?
1

they occur as a result of increased elastic tone
2

they lie parallel to the underlying muscles
3

the long axis of a wound often lies in the direction of the RSTL
4

incisional scars should be placed perpendicular to RSTLs
31

5

smiling minimizes the appearance of RSTLs
Q/Q(M)-474745 Report a Problem
Which statement is true regarding relaxed skin tension lines (RSTLs)?
3

the long axis of a wound often lies in the direction of the RSTL
Relaxed skin tension lines are creases in the skin that are present at rest and develop as a consequence of
the decreased elasticity that occurs with aging and solar damage. These lines lie perpendicular to the
underlying musculature and can be accentuated by smiling or frowning. An undermined circular wound
will often form an oval shape due to muscular tension, and will have its long axis oriented within these
relaxed tension lines. Knowledge of RSTLs is important in cutaneous surgery because placing incisions
within these lines will ensure the most favorable cosmetic result.
Q/Q(M)-474745 Report a Problem


What is the tattoo pigment responsible for most lichenoid reactions?
1

Titanium dioxide
2

Carbon
3

Mercuric sulfide
4

Chromates
5

Iron oxide
Q/Q(M)-476765 Report a Problem

What is the tattoo pigment responsible for most lichenoid reactions?
3

Mercuric sulfide
Lichenoid tattoo reactions are not as common as eczematous reactions, both of which are most
commonly caused by mercuric sulfide. Lichenoid reaction are likely to be mediated by delayed
hypersensitivity to a lymphocytic T-cell infiltrate.
Q/Q(M)-476765 Report a Problem


Which of the following treatment modalities for basal cell carcinoma has the highest long term cure
rate?
1

MOHS surgery
2

Radiation
3

Cryotherapy
4

Electrodessication and curettage
5

Surgical excision
Q/Q(M)-482825 Report a Problem

Which of the following treatment modalities for basal cell carcinoma has the highest long term cure
rate?
1

MOHS surgery
32

Five year cure rates of basal cell carcinomas (BCC) treated with MOHS is about 99%. The other options
listed range from 90-92% five year cure rates.
Q/Q(M)-482825 Report a Problem

A history of anaphylaxis is a contraindication to treatment with which cosmetic injectable?
1

Sculptra
2

Myobloc
3

Radiance
4

Cosmoderm
5

Restylane
Q/Q(M)-475347 Report a Problem


A history of anaphylaxis is a contraindication to treatment with which cosmetic injectable?
5

Restylane
Restylane is a hyaluronic acid gel produced by the Streptococcus species of bacteria. The package insert
for Restylane warns that its use is contraindicated in patients with severe allergies manifested by a
history of anaphylaxis or the presence of multiple severe allergies.
Q/Q(M)-475347 Report a Problem



Compared to intact skin, what is the tensile strength of a wound 6 months after surgery?
1

30%
2

50%
3

70%
4

90%
5

100%
Q/Q(M)-477178 Report a Problem



Compared to intact skin, what is the tensile strength of a wound 6 months after surgery?
3

70%
Tissue remodeling allows the host to develop a scar that is approximately 70% of the original strength of
the skin.
Q/Q(M)-477178 Report a Problem



What risk does an organ transplant recipient have of developing melanoma?
1

Same risk as general population
2

Less risk than general population
33

3

3-fold increased risk
4

10- fold increased risk
5

50-fold increased risk
Q/Q(M)-475365 Report a Problem


What risk does an organ transplant recipient have of developing melanoma?
3

3-fold increased risk
Organ transplant recipients are at risk for developing many cutaneous tumors. The risk of melanoma is
3-fold that of the general population whereas these patients have a 65-fold increased risk of developing
squamous cell carcinoma.
Q/Q(M)-475365 Report a Problem
Which of the following lasers may cause milia formation as a post-procedure complication?
1

Excimer
2

KTP
3

Pulsed Dye
4

ND-Yag
5

Erbium
Q/Q(M)-476775 Report a Problem


Which of the following lasers may cause milia formation as a post-procedure complication?
5

Erbium
The erbium (2940 nm) and carbon dioxide (10,600 nm) lasers may both cause milia formation after laser
skin resurfacing.
Q/Q(M)-476775 Report a Problem


Which of the following topical antibacterial agents may cause neutropenia?
1

Mupirocin
2

Silver sulfadiazine
3

Polymyxin
4

Bacitracin
5

Povodine-iodine
Q/Q(M)-474733 Report a Problem

Which of the following topical antibacterial agents may cause neutropenia?
2

Silver sulfadiazine
Silver sulfadiazine is most commonly used to prevent infection in second and third-degree burn patients.
Rare cases of leukopenia, neutropenia and kernicterus have been reported in patients using this topical
antibacterial agent. Consequently, silver sulfadiazine should be used cautiously and avoided in infants,
34

nursing mothers and pregnant women. Silver sulfadiazine also has the potential to cause a
hypersensitivity reaction in patients with sulfa allergy.
Q/Q(M)-474733 Report a Problem


The most appropriate laser to treat a port wine stain on an infants cheek would have which of the
following wavelengths?
1

488 nm
2

532 nm
3

585 nm
4

694 nm
5

810 nm
Q/Q(M)-474718 Report a Problem

The most appropriate laser to treat a port wine stain on an infants cheek would have which of the
following wavelengths?
3

585 nm
The 585nm pulsed dye laser (PDL) targets intravascular oxyhemoglobin and is considered the treatment
of choice for most benign vascular lesions. The original PDL had a wavelength of 577 nm which was
later modified to 585 nm to achieve deeper penetration yet still maintainvascular specificity.
Q/Q(M)-474718 Report a Problem


A patient with end stage liver disease comes to your surgical practice. Which of the following
anesthetics should be used with this patient?
1

Benzocaine
2

Etidocaine
3

Lidocaine
4

Mepivicaine
5

Prilocaine
Q/Q(M)-477388 Report a Problem

A patient with end stage liver disease comes to your surgical practice. Which of the following
anesthetics should be used with this patient?
1

Benzocaine
Benzocaine, an ester, should be used as it is the only anesthetic mentioned above that is metabolized by
the kidney and not the liver.
Q/Q(M)-477388 Report a Problem



A physician using a carbon dioxide laser to treat verruca should be aware that all of the following
complications can occur except:
35

1

Ocular damage
2

Transmission of viral disease
3

Purpura
4

Recurrence of lesion
5

Scarring
Q/Q(M)-477211 Report a Problem



A physician using a carbon dioxide laser to treat verruca should be aware that all of the following
complications can occur except:
3

Purpura
The carbon dioxide laser uses a 10,600 nm wavelength to target water as a chromophore. Lasers that
target water may damage the cornea. The carbon dioxide laser can be used to destroy epidermal and
dermal lesions such as warts. HPV virus has been recovered in the laser plume after treatment with
carbon dioxide laser. Bovine papilloma virus recovered in the plume was even found to transmit disease
to calf skin. Scarring and recurrence have also been found to be potential complications.
Q/Q(M)-477211 Report a Problem


The anatomic structure identified in the photograph is called the:
1

Triangular fossa
2

Concha
3

Antihelix
4

Scaphoid fossa
5

Tragus
Q/Q(M)-475478 Report a Problem



The anatomic structure identified in the photograph is called the:
36

1

Triangular fossa
The triangular fossa of the ear is shown in the photograph. This anatomic structure is bordered by the
crura of the antihelix.
Q/Q(M)-475478 Report a Problem


In organ transplant recipients, all of the following factors increase susceptibility to the development of
skin cancer except:
1

Fair skin (Fitzpatrick types I-III)
2

History of chronic sun exposure
3

Duration of immunosuppression
4

History of HPV infection
5

CD8 lymphocytopenia
Q/Q(M)-475366 Report a Problem


In organ transplant recipients, all of the following factors increase susceptibility to the development of
skin cancer except:
5

CD8 lymphocytopenia
All organ transplant recipients are at increased risk for the development of cutaneous malignancies.
However, the above-mentioned factors, with the exception of choice E, place these individuals at further
risk. CD4 lymphocytopenia, rather than CD8, is another identified risk factor as is older age, history of
actinic keratoses and history of skin cancer.
Q/Q(M)-475366 Report a Problem


A safe dose of lidocaine with epinephrine when used in tumescent anesthesia is:
1

3 mg/kg
2

4.5 mg/kg
3

7 mg/kg
4

20 mg/kg
5

50 mg/kg
Q/Q(M)-477369 Report a Problem


A safe dose of lidocaine with epinephrine when used in tumescent anesthesia is:
5

50 mg/kg
The maximum recommended dosage of lidocaine in adults is 4.5 mg/kg without epinephrine, 7.0 mg/kg
with epinephrine, and 55 mg/kg in tumescent anesthesia for liposuction.
Q/Q(M)-477369 Report a Problem


Which of the following sterilization methods has the negative side effect of dulling instruments?
1

chemical autoclave
37

2

steam autoclave
3

dry heat
4

cold sterilization
5

gas sterilization
Q/Q(M)-482910 Report a Problem


Which of the following sterilization methods has the negative side effect of dulling instruments?
2

steam autoclave
Steam autoclave is the most common type of sterilization used in the office setting. It is not to be used
with heat-sensitive plastics. A negative side effect of using steam autoclave is that it can dull sharp
instruments.
Q/Q(M)-482910 Report a Problem

Which anesthetic has the shortest duration of action?
1

Mepivacaine
2

Procaine
3

Bupivacaine
4

Tetracaine
5

Prilocaine
Q/Q(M)-477234 Report a Problem

Which anesthetic has the shortest duration of action?
2

Procaine
Procaine has the shortest duration of action, only lasting about 30-60 minutes. It is an ester anesthetic
and is commonly used in dentistry.
Q/Q(M)-477234 Report a Problem


The relaxed skin tension lines overlying the scapula are:
1

Do not exist over the scapula
2

Oriented vertically
3

Oriented circumferentially
4

Oriented horizontally
5

Parallel to the underlying muscle groups
Q/Q(M)-475349 Report a Problem



The relaxed skin tension lines overlying the scapula are:
2

Oriented vertically
38

Relaxed skin tension lines (RSTLs) are creases in the skin that are present at rest and develop as a
consequence of the decreased elasticity that occurs with aging and solar damage. These lines lie
perpendicular to the underlying musculature. They tend to run vertically on the upper back whereas on
the central trunk, they are typically circumferential. Knowledge of RSTLs is important in cutaneous
surgery because placing incisions within these lines will ensure the most favorable cosmetic result.
Q/Q(M)-475349 Report a Problem


Which of the following is true regarding lidocaine:
1

1% lidocaine is equal to 1g/10ml
2

Duration with no epinephrine is 4-6 hours
3

Maximum dose with no epinephrine is 7mg/kg
4

Beta blockers increase lidocaine levels
5

Allergy most commonly occurs to propylene glycol preservatives
Q/Q(M)-477476 Report a Problem



Which of the following is true regarding lidocaine:
4

Beta blockers increase lidocaine levels
Beta blockers increase lidocaine levels. The rest of the answers are false. 1% lidocaine is equal to
1g/100ml or 10mg/ml. Duruation of lidocaine with no epi is 30-60 minutes. Maximum dose of lidocaine
with no epi is 4.5 mg/kg, with epi it is 7 mg/kg. Allergy to lidocaine is most commonly due to paraben
preservatives.
Q/Q(M)-477476 Report a Problem


Formation of granulomas is a potential complication of treatment with:
1

Isotretinoin
2

Autologous fat
3

CO2 resurfacing
4

Silicone
5

Hydroxychloroquine
Q/Q(M)-474751 Report a Problem


Formation of granulomas is a potential complication of treatment with:
4

Silicone
Silicone is a synthetic, viscous compound which is composed of long polymers of dimethylsiloxanes.
Silikon-1000 is one silicone product which is available in the United States and is FDA-approved for the
ophthalmic treatment of complicated retinal detachment. The number 1000 refers to the products
viscosity which is measured in centistokes. As a reference, water has a viscosity of 100 centistokes and
mineral oil has a viscosity of about 350 centistokes. With respect to cosmetic applications, silicone is not
FDA-approved for soft tissue augmentation. In addition to the occurrence of hypersensitivity reactions
and product migration, there have been many reports of granuloma formation after silicone injection,
even many years post-treatment.
39

Q/Q(M)-474751 Report a Problem


Full thickness skin grafts are more likely than split thickness skin grafts to demonstrate:
1

Alopecia
2

Contraction
3

Necrosis
4

Hypohidrosis
5

Hypopigmentation
Q/Q(M)-482350 Report a Problem


Full thickness skin grafts are more likely than split thickness skin grafts to demonstrate:
3

Necrosis
Full thickness skin grafts have more adnexal structures, vessels, and structures to supply. Therefore, they
are more prone to necrosis. However, they are less prone to contraction, alopecia, hypohidrosis, and
hypopigmentation for similar structural reasons.
Q/Q(M)-482350 Report a Problem

Which material is used to coat the jaws of the needle holder in the photograph?
1

Gold
2

Tungsten carbide
3

Copper
4

Stainless steel
5

Nickel
Q/Q(M)-475485 Report a Problem



Which material is used to coat the jaws of the needle holder in the photograph?
2

Tungsten carbide
Tungsten carbide is believed to be a harder more durable material that allows for better grasping of the
needle. Needle holders with tungsten carbide jaws usually have gold-plated handles.
Q/Q(M)-475485 Report a Problem


40


Which of the following histochemical stains can be used intraoperatively during Mohs micrographic
surgery to confirm the diagnosis of extramammary Paget's disease?
1

Neuron-specific enolase
2

Toluidine blue
3

S-100
4

Cytokeratin 7
5

Myeloperoxidase
Q/Q(M)-474734 Report a Problem


Which of the following histochemical stains can be used intraoperatively during Mohs micrographic
surgery to confirm the diagnosis of extramammary Paget's disease?
4

Cytokeratin 7
Cytokeratin 7 is a structural component of the cytoskeleton that stains poorly differentiated tumors of
the epithelium. This low molecular weight cytokeratin positively stains Paget cells. Neuron-specific
enolase stains merkel cell carcinoma, toluidine blue stains mast cells and myeloperoxidase stains cells
with myeloid differentiation such as leukemia cells. S-100 protein is a non-specific stain that is
commonly used as an adjunctive marker in the diagnosis of melanoma.
Q/Q(M)-474734 Report a Problem

Which of the following is true regarding a Baker-Gordon peel?
1

88% phenol is the most important component for efficacy
2

It contains salicytic acid
3

It contains resorcinol
4

It is considered a medium depth peel
5

Croton oil is considered the most important component for efficacy
Q/Q(M)-477477 Report a Problem


Which of the following is true regarding a Baker-Gordon peel?
5

Croton oil is considered the most important component for efficacy
Croton oil is considered the most important component for efficacy. It is a epidermolytic that enhances
penetration. A Baker Gordon peel is a deep peel that contains 88% phenol, tap water, croton oil, and
septisol.
Q/Q(M)-477477 Report a Problem


The tissue temperature required for destruction of basal cell or squamous cell skin cancers with
cryotherapy:
1

-40 to 50 C
2

0 to -10 C
41

3

10 to 20 C
4

-60 to- 70 C
5

-10 to -20 C
Q/Q(M)-477414 Report a Problem

The tissue temperature required for destruction of basal cell or squamous cell skin cancers with
cryotherapy:
1

-40 to 50 C
-40 to -50 C is the temperature required in the treated skin to effectively destroy basal cell and
squamous cell skin cancers. To treat malignant skin lesions a thermocoupling device can be used to
measure the exact temperature of the skin while freezing. After anesthetizing the skin, this device is
inserted into the estimated depth of the skin lesion. Cryotherapy is then performed until the thermometer
reaches a temperature of approximately -50 degrees Celsius. Keratinocytes need to be frozen to -50
degrees Celsius for effective destruction, while melanocytes are destroyed at -5 degrees Celsius. To treat
cutaneous malignancies,-50 Degress Celsius is the effective temperature. For benign skin lesions such as
verrucae, -20 degrees Celsius is effective.
Q/Q(M)-477414 Report a Problem



Which of the following medicines has a newly reported side effect of eruptive epidermoid cysts?
1

Minocycline
2

Retinoids
3

Imiquimod
4

Denileukin difitox
5

Potassium iodide
Q/Q(M)-482243 Report a Problem

Which of the following medicines has a newly reported side effect of eruptive epidermoid cysts?
3

Imiquimod
Imiquimod is a topical immunomodulator which induces the toll like receptor 7 and is used in the
treatment of genital warts, actinic keratoses, and superficial basal cell carcinomas. It has a newly
reported side effect of eruptive epidermoid cysts. Minocycline has side effects including a lupus-like
syndrome and hepatitis. Retinoids has numerous side effects including hypertriglyceridemia, elevated
liver enzymes, dryness of skin, teratogenicity, and most recently inflammatory bowel disease.
Denileukin difitox, an engineered protein combining interleukin-2 and Diphtheria toxin, used in mycosis
fungoides may cause vascular leak syndrome characterized by hypotension, edema, and pleural effusion.
Potassium iodide may lead to the Wolff-Chaikoff effect in which thyroid hormone synthesis is inhibited
from excess iodides that block organic iodides from binding in the thyroid.
Q/Q(M)-482243 Report a Problem

Which part of the eye is most likely to be damaged to exposure to a pulsed dye laser?
1

Retina
42

2

Cornea
3

Lens
4

Conjuctiva
5

Sclera
Q/Q(M)-476774 Report a Problem



Which part of the eye is most likely to be damaged to exposure to a pulsed dye laser?
1

Retina
The pulsed dye laser operates at 585-nm and targets hemoglobin as a chromophore. It can pass through
the cornea and damage choroidal and retinal vasculature. Several infrared pigment-specific lasers (eg,
ruby, alexandrite, Nd:YAG) may also damage the retina by targeting the retinal pigment.
Q/Q(M)-476774 Report a Problem

Pre-testing for allergy to collagen is required for which cosmetic filler?
1

Radiance
2

Artecoll
3

Perlane
4

Cosmoderm
5

Silicone
Q/Q(M)-474715 Report a Problem

Pre-testing for allergy to collagen is required for which cosmetic filler?
2

Artecoll
Artecoll is a nonbiodegradable injectable filler composed of microspheres of polymethylmethacrylate
which are suspended in bovine collagen. As with other types of injectable bovine collagen, patients must
be tested for allergy to bovine collagen prior to treatment.
Q/Q(M)-474715 Report a Problem


A patient with a deep vertical frown line in the glabellar region requests Botox. Treatment will mostly
target which one of the following muscles?
1

Oculi
2

Orbicularis oculi
3

Depressor superciliaris
4

Levator palpebrae superioris
5

Corrugator superciliaris
Q/Q(M)-474721 Report a Problem


43

A patient with a deep vertical frown line in the glabellar region requests Botox. Treatment will mostly
target which one of the following muscles?
5

Corrugator superciliaris
Although the procerus and orbicularis oculi muscles are contributory, the development of vertical
glabellar frown lines is mostly due to the overactivity of the corrugator supercilaris. These muscles
function to pull the eyebrow inferomedially.
Q/Q(M)-474721 Report a Problem

Which ocular structure is at most risk of injury from an erbium:YAG laser?
1

Lens
2

Retina
3

Cornea
4

Iris
5

Pupil
Q/Q(M)-474728 Report a Problem

Which ocular structure is at most risk of injury from an erbium:YAG laser?
3

Cornea
The erbium:YAG laser is an ablative laser whose target chromophore is water. The cornea is an aqueous
structure and is therefore the most susceptible to damage from this laser.
Q/Q(M)-474728 Report a Problem


What is the ratio of sodium bicarbonate to 1% lidocaine in a buffered lidocaine solution?
1

1:1
2

1:2
3

1:5
4

1:10
5

1:25
Q/Q(M)-477132 Report a Problem


What is the ratio of sodium bicarbonate to 1% lidocaine in a buffered lidocaine solution?
4

1:10
The use of sodium bicarbonate in 1% lidocaine reduces the acidity of the local anesthetic. Unbuffered
lidocaine has pH of approximately 5.5 -7.0 and lidocaine with epinephrine has a pH of approximately
3.5 5.0. The addition of sodium bicarbonate at a ratio 1:10 raises the pH to a more physiologic level,
thereby reducing the stinging sensation associated with injection. However, the addition of bicarbonate
also reduces the shelf-life of the lidocaine.
Q/Q(M)-477132 Report a Problem
The main advantage of selecting 4-0 Vicryl rather than 5-0 Vicryl to suture a wound is:
44

1

Smaller suture diameter
2

Increased tensile strength
3

Increased knot security
4

Smaller needle
5

Increased suture memory
Q/Q(M)-474735 Report a Problem

The main advantage of selecting 4-0 Vicryl rather than 5-0 Vicryl to suture a wound is:
2

Increased tensile strength
Sutures are classified according to the United States Pharmacopeia (USP) criteria. This classification
system specifies the diameter of a given suture material that is required to produce a certain tensile
strength. The smaller the cross-sectional diameter of a suture material, the higher the USP number that is
assigned. Thus, 4-0 Vicryl will have greater tensile strength and a larger cross-sectional diameter than 5-
0 Vicryl.
Q/Q(M)-474735 Report a Problem

Which porphyrin is targeted in aminolevulinic acid-photodynamic therapy?
1

Protoporphyrin IX
2

Uroporphyrinogen
3

Coproporphyrin III
4

Coproporphyrinogen
5

Porphobilinogen
Q/Q(M)-476512 Report a Problem


Which porphyrin is targeted in aminolevulinic acid-photodynamic therapy?
1

Protoporphyrin IX
Protoporphyrin IX is the substrate for the final rate-limiting step of heme synthesis. This enzymatic
reaction is catalyzed by ferrochelatase. The preferential uptake of aminolevulinic acid by cancerous and
pre-cancerous cells results in higher concentrations of protoporphyrin IX. This can then be selectively
targeted by photodynamic therapy.
Q/Q(M)-476512 Report a Problem


The use of EMLA cream is contraindicated in patients with which of the following?
1

Atopic dermatitis
2

Deomycin allergy
3

Sickle cell anemia
4

Methemoglobinemia
5

Peripheral neuropathy
Q/Q(M)-477147 Report a Problem
45


The use of EMLA cream is contraindicated in patients with which of the following?
4

Methemoglobinemia
The most serious adverse effect of eutectic lidocaine and prilocaine (EMLA) is methemaglobinemia. A
metabolite of prilocaine oxidizes hemoglobin to methemoglobin, which is less efficient in release of
oxygen leading to tissue hypoxia. Patients with congenital or idiopathic methemoglobinemia or infants
under 1 year of age are at higher risk for the development of this side effect.
Q/Q(M)-477147 Report a Problem

The Shaw scalpel is used:
1

As a form of electrocautery
2

To enhance cyst wall removal
3

For incision and drainage
4

For thicker tissue (back)
5

For delicate work around the eyes and ears
Q/Q(M)-477419 Report a Problem


The Shaw scalpel is used:
1

As a form of electrocautery
The Shaw scalpel provides the ability to cut through tissue with minimal blood loss. This instrument
employs an elctronically heated metal cutting balde that heats up while it cuts through tissue, and
coagulates fine vessels when they are divided. This instrument has been used frequently for treatment of
rhinophyma and in parotid gland surgeries.
Q/Q(M)-477419 Report a Problem
While contemplating the repair of a large cheek defect after Mohs surgery, you begin to anesthetize an
elderly lady who weighs 110 lbs. What is the maximum amount of 1% lidocaine with 2.5% epinephrine
that this patient can receive?
1

10 ml
2

35 ml
3

50 ml
4

350 ml
5

500 ml
Q/Q(M)-474719 Report a Problem


While contemplating the repair of a large cheek defect after Mohs surgery, you begin to anesthetize an
elderly lady who weighs 110 lbs. What is the maximum amount of 1% lidocaine with 2.5% epinephrine
that this patient can receive?
2

35 ml
The maximum recommended dosage of 1% lidocaine with 2.5% epinephrine in adults is 7mg/kg. The
patient weighs 110 lbs (or 50 kg) which allows her 350 mg or 35 ml (1% lidocaine has 10mg of
46

lidocaine perml) of the anesthetic.
Q/Q(M)-474719 Report a Problem


Topical vitamin K has been shown to:
1

Increases epidermal differentiation
2

Reduce severity of postoperative purpura
3

Decrease epidermal pigmentation
4

Improve fine winkling
5

Improve skin hydration
Q/Q(M)-477182 Report a Problem


Topical vitamin K has been shown to:
2

Reduce severity of postoperative purpura
Topical vitamin K reduced post later treatment purpura when applied after (not before) treatment of
telangectasia with the 585 nm pulsed dye laser.
Q/Q(M)-477182 Report a Problem



Which of the following immunosuppressive agents may exert a protective effect against the
development of skin cancers in organ transplant patients?
1

Cyclosporine
2

Rapamycin
3

Tacrolimus
4

Corticosteroids
5

Azathioprine
Q/Q(M)-475360 Report a Problem


Which of the following immunosuppressive agents may exert a protective effect against the
development of skin cancers in organ transplant patients?
2

Rapamycin
Rapamycin (also called sirolimus) is a macrolide antibiotic and a structural analog of FK 506. It is a
potent immunosuppressive agent which inhibits mTOR (a member of P13K family kinases). Despite its
immunosuppressive effects, preliminary data show a decreased incidence of skin cancer in organ
transplant patients treated with rapamycin and postulate that it may exert a protective effect against
cutaneous malignancies.
Q/Q(M)-475360 Report a Problem


The anatomic structure identified in the photograph is called the:
1

Soft triangle
47

2

Columella
3

Cupids bow
4

Philtral ridge
5

Vermillion border
Q/Q(M)-475479 Report a Problem



The anatomic structure identified in the photograph is called the:
4

Philtral ridge
The philtral ridges are located at the upper middle lip and form part of the aesthetically important
cupids bow.
Q/Q(M)-475479 Report a Problem



All of the following statements are true regarding the MCW Melanoma Cocktail except:
1

it is a mixture of monoclonal antibodies to MART-1
2

it is performed intraoperatively
3

it is a mixture of monoclonal antibodies to Melan-A
4

it demonstrates micrometastases in sentinel lymph nodes
5

it is a mixture of polyclonal antibodies to tyrosinase
Q/Q(M)-475361 Report a Problem


All of the following statements are true regarding the MCW Melanoma Cocktail except:
5

it is a mixture of polyclonal antibodies to tyrosinase
The MCW Melanoma Cocktail is an immunostain made up of monoclonal antibodies to MART-1,
Melan-A and tryrosine. It is used intraoperatively during sentinel lymph node biopsy and allows for
rapid and accurate determination of micrometastases. Shidham VB et al. Optimization of an
immunostaining protocol for the rapid intraoperative evaluation of melanoma sentinel lymph node
imprint smears with the MCW Melanoma Cocktail. Cytojournal 2004;1(1):2.
Q/Q(M)-475361 Report a Problem

Which tattoo pigment has been most commonly associated with pseudolymphomatous reactions?
48

1

Mercuric sulfide
2

Cadmium sulfide
3

Phthalocyanine dyes
4

Malachite
5

Cobalt
Q/Q(M)-476766 Report a Problem



Which tattoo pigment has been most commonly associated with pseudolymphomatous reactions?
1

Mercuric sulfide
Some delayed type hypersensitivity reactions may lead to pseudolymphoma, most commonly caused by
red tattoo pigment (mercuric sulfide).
Q/Q(M)-476766 Report a Problem

The use of imiquimod (Aldara) for the treatment of superficial basal cell carcinoma is advocated at
which treatment regimen?
1

Five times a week for 6 weeks
2

Five times per week for 4 weeks
3

Three times a week for 6 weeks
4

Three times per week for 4 weeks
5

Three times a week for 10 weeks
Q/Q(M)-477056 Report a Problem
The use of imiquimod (Aldara) for the treatment of superficial basal cell carcinoma is advocated at
which treatment regimen?
1

Five times a week for 6 weeks
Imiquimod (Aldara) was FDA-approved in 2004 for the treatment of superficial basal cell carcinoma.
The recommended treatment schedule is once daily, five times per week for a total of six weeks. Geisse
J et al. Imiquimod 5% cream for the treatment of superficial basal cell carcinoma: results from two
phase III, randomized, vehicle-controlled studies.
Q/Q(M)-477056 Report a Problem


The classic purpose for serrated scissors is:
1

For work on thicker tissues (back)
2

For work on delicate areas around the eyes and the ears
3

For gripping tissue while cutting
4

For tissue undermining
5

For sharp dissections of tissue
Q/Q(M)-477420 Report a Problem
49



The classic purpose for serrated scissors is:
3

For gripping tissue while cutting
Serrated scissors are used primarily for gripping tissue to prevent sliding while cutting.
Q/Q(M)-477420 Report a Problem


The anatomic structure identified in the photograph is called the:
1

Nasal ala
2

Soft triangle
3

Columella
4

Philtral ridge
5

Nasal groove
Q/Q(M)-475480 Report a Problem


The anatomic structure identified in the photograph is called the:
3

Columella
The columella is one of the cosmetic subunits of the nose. It is formed by the medial crura of the alar
cartilage.
Q/Q(M)-475480 Report a Problem



Which of the following sutures is the first to be absorbed?
1

Catgut
2

Polyglycolic acid
3

Polyglactin 910
4

Polydioxanone
5

Polypropylene
Q/Q(M)-475354 Report a Problem
Which of the following sutures is the first to be absorbed?
1

Catgut
50

Catgut has a variable rate of absorption but typically lasts about 7-14 days. Thus, it would be the first
suture to be absorbed. Polyglactin 910 (Vicryl) and polyglycolic acid (Dexon) are both absorbed in
about 90 days. Polydioxanone (PDS) is an absorbable monofilament which lasts approximately 180
days. Polypropylene (Prolene) is not an absorbable suture.
Q/Q(M)-475354 Report a Problem


Endocarditis prophylaxis is recommended for each of the following except:
1

Prosthetic valves
2

Atrial septal defects
3

Hypertrophic cardiomyopathy
4

Mitral valve prolapse with regurgitation
5

Previous bacterial endocarditis
Q/Q(M)-477677 Report a Problem
Endocarditis prophylaxis is recommended for each of the following except:
2

Atrial septal defects
Preoperative antibiotics are recommended for endocarditits prophylaxis in those patients with prostheitc
valves, congenital cardiac malformations, previous bacterial endocarditis, rheumatic heart disease with
valve dysfunction, hypeertrophic cardiomyopathy, and mitral valve prolapse with refurgitaion.
Classically 1 gram of Dicloxacillin or cephalexin is given 1 hour preoperatively and an additional 500
mg is given 6 hours post op. Clindamycin can be givenin those patients who are penicillin allergic.
Preopertive antibiotics for endocarditis prohpylaxis are not required in those patients who have an Atrial
Septal Defect (ASD). pacemaker, rheumatic fever without valve dysfunction, or history of Coronary
Artery Bypass Graft (CABG).
Q/Q(M)-477677 Report a Problem


Which of the following facial rejuvenation techniques creates microthermal zones?
1

Radiofrequency
2

CO2 laser
3

Intense pulsed light
4

Dermabrasion
5

Fractional photothermolysis
Q/Q(M)-475363 Report a Problem


Which of the following facial rejuvenation techniques creates microthermal zones?
5

Fractional photothermolysis
Fractional photothermolysis utilizes an infrared laser to create thousands of microthermal zones with
normal skin remaining between these zones. The zones of normal skin support rapid re-epithelialization
and result in much faster healing times than with traditional resurfacing procedures.
Q/Q(M)-475363 Report a Problem


Activation of the procerus muscle causes
51

1

Periocular wrinkles
2

Wrinkles at the nasal root
3

Wrinkles on the forehead
4

Perioral wrinkles
5

Accentuation of the melolabial folds
Q/Q(M)-477417 Report a Problem


Activation of the procerus muscle causes
2

Wrinkles at the nasal root
The procerus muscle causes wrinkling at the nasal root and is often targeted with Botox therapy for
improved cosmesis.
Q/Q(M)-477417 Report a Problem



The following are true statements regarding the design of an excisional ellipse except:
1

Ellipse includes pathologic and normal tissue
2

Length of ellipse is at least 3 times its total width
3

The angle at each of the apices is 15
4

Incision lines are placed in relaxed skin tension lines
5

Long axis of ellipse is oriented perpendicular to free margins
Q/Q(M)-474982 Report a Problem



The following are true statements regarding the design of an excisional ellipse except:
3

The angle at each of the apices is 15
Thoughtful design of an elliptical excision is important for obtaining optimum tissue diagnosis, tumor
clearance, wound closure and cosmetic result. The above statements are all true except that the angles at
the apices of the ellipse should be 30.
Q/Q(M)-474982 Report a Problem



The O to Z flap is what type of flap?
1

Rotation
52

2

Advancement
3

Transposition
4

Pedicle
5

Bilobed
Q/Q(M)-477392 Report a Problem

The O to Z flap is what type of flap?
1

Rotation
The O to Z flap is a bilateral rotation flap in which tissue is moved from two ends to cover a central
defect. It is often used on the scalp or lower extremities in locations where tissue laxity is not present.
Q/Q(M)-477392 Report a Problem

What nerve innervates the skin between the 1st and 2nd toes?
1

saphenous nerve
2

posterior tibial nerve
3

sural nerve
4

superficial peroneal nerve
5

deep peroneal nerve
Q/Q(M)-482915 Report a Problem


What nerve innervates the skin between the 1st and 2nd toes?
5

deep peroneal nerve
deep peroneal nerve innervates the skin between the 1st and 2nd toes. The saphenous innervates the
instep and medial ankle. The posterior tibial innervates the heel and middle sole of the foot. The sural
nerve innervates the 5th toe and lateral side of the sole. The superficial peroneal nerve innervates the
skin of the toes other than the outside of the 5th toe and in between the 1st and 2nd toes.
Q/Q(M)-482915 Report a Problem


Which type of flap moves almost entirely in one direction in a linear fashion
1

Rhombic flap
2

Advancement flap
3

Rotation flap
4

Transposition flap
5

Bilobed flap
Q/Q(M)-478769 Report a Problem


Which type of flap moves almost entirely in one direction in a linear fashion
2

Advancement flap
53

An advancement flap is one that moves almost entirely in one direction or in a linear fashion. Rotation
flap alters the direction of major closure tension forces. Transposition flap redirects and redistributes
tension. Rhombic and bilobed flaps are both examples of transposition flaps.
Q/Q(M)-478769 Report a Problem


Injection of Botox at the location identified in the photograph would denervate which muscle?
1

Orbicularis oculi
2

Frontalis
3

Corrugator superciliaris
4

Procerus
5

Levator palpebrae superioris
Q/Q(M)-475487 Report a Problem



Injection of Botox at the location identified in the photograph would denervate which muscle?
4

Procerus
The development of vertical glabellar frown lines is due to the overactivity of the orbicularis oculi,
procerus and corrugator supercilaris muscles. These muscles function to pull the eyebrow
inferomedially.
Q/Q(M)-475487 Report a Problem



The cutaneous lip and chin are divided into cosmetic units by which anatomic boundary?
1

Vermillion border
2

Nasolabial fold
3

Marionette lines
4

Mental crease
5

Philtral ridge
Q/Q(M)-475350 Report a Problem


The cutaneous lip and chin are divided into cosmetic units by which anatomic boundary?
4

Mental crease
54

The term cosmetic unit is used to define regions of the face by specific characteristics such as color,
texture, presence of hair, and prominence of sebaceous glands. In dermatologic surgery, it is preferable
to conceal incisions within the boundary lines of cosmetic units. The mental crease divides the
cutaneous lip and chin.
Q/Q(M)-475350 Report a Problem


Which potential complications is seen exclusively with phenol peels?
1

Milia
2

Hypopigmentation
3

Toxic shock syndrome
4

Prolonged erythema
5

Laryngeal edema
Q/Q(M)-474750 Report a Problem


Which potential complications is seen exclusively with phenol peels?
5

Laryngeal edema
The principle concept of chemical peeling is to utilize a chemical agent to produce a controlled wound
in the skin. Many different types of side effects can occur with chemical resurfacing and are typically
related to the depth of the wound. Complications which are common to all peeling agents include: milia;
acne; pigmentary changes; contact dermatitis; scarring; infection; prolonged erythema; textural changes
and cold sensitivity. Unlike other peeling agents, phenol has the potential to cause severe adverse
reactions which are exclusive to its use only. They include atrophy, cardiac arrhythmias and laryngeal
edema.
Q/Q(M)-474750 Report a Problem
When would use of this technique be most beneficial?
1

To prevent post-operative bleeding
2

To prevent post-operative infection
3

To shorten the length of the wound
4

To increase the width of the wound
5

To prevent pincushioning
Q/Q(M)-476760 Report a Problem
55


When would use of this technique be most beneficial?
3

To shorten the length of the wound
M-plasty is used to shorten wound length and is often when it is not desirable to extend the ellipse into a
nearby structure.
Q/Q(M)-476760 Report a Problem



Which of the following is the earliest symptom of lidocaine toxicity:
1

Tachycardia
2

Perioral tingling
3

Nystagmus
4

Cyanosis
5

Seizure
Q/Q(M)-477117 Report a Problem

Which of the following is the earliest symptom of lidocaine toxicity:
2

Perioral tingling
The maximum dosage of lidocaine is 4.5 mg/kg without epinephrine and 7.0 mg/kg with epinephrine.
Signs of lidocaine toxicity start at 1-5 microgm/ml with increased anxiety, talkativeness, tinnitus,
numbess/tingling around lips, metallic taste, double vision. Higher levels of toxicity may cause
nystagmus, muscle twitching, tremor and finally seizures and respiratory arrest.
Q/Q(M)-477117 Report a Problem
Which tattoo pigment is most commonly associated with allergic reactions, eczematous and
granulomatous?
1

Mercuric sulfide
2

Titanium dioxide
3

Cadmium sulfide
4

Cobalt
56

5

Carbon
Q/Q(M)-476763 Report a Problem
Which tattoo pigment is most commonly associated with allergic reactions, eczematous and
granulomatous?
1

Mercuric sulfide
Allergic reactions have been reported with several different types of tattoo pigment. The most
commonly associated tattoo pigment however is mercuric sulfide. Tattoo with mercuric sulfide produces
a red color.
Q/Q(M)-476763 Report a Problem
The rhytids identified in the photograph are referred to as:
1

Marionette lines
2

Relaxed skin tension lines
3

Crows feet
4

Nasolabial folds
5

Bunny lines
Q/Q(M)-475481 Report a Problem


The rhytids identified in the photograph are referred to as:
5

Bunny lines
Bunny lines are located on the dorsum of the nose and are formed by the contraction of the nasalis
muscle.
Q/Q(M)-475481 Report a Problem


Which of the following is a vasoconstrictor in the absence of epinephrine?
1

Lidocaine
2

Procaine
3

Cocaine
4

Bupivacaine
5

Prilocaine
Q/Q(M)-482916 Report a Problem


57


Which of the following is a vasoconstrictor in the absence of epinephrine?
3

cocaine
Cocaine is the only anesthetic that vasoconstricts in the absence of epinephrine. Cocaine is an ester
anesthetic. Full vasoconstriction effects of epinephrine requires 7-15 minutes. Epinephrine is pregnancy
category C.
Q/Q(M)-482916 Report a Problem

Which tattoo pigment has most commonly been associated with phototoxic reactions?
1

Titanium dioxide
2

Mercuric sulfide
3

Carbon
4

Iron oxide
5

Cadmium sulfide
Q/Q(M)-476764 Report a Problem

Which tattoo pigment has most commonly been associated with phototoxic reactions?
5

Cadmium sulfide
Cadmium sulfide produces a yellow tattoo. Phototoxic reactions are most commonly caused by
cadmium sulfide tattoo pigment. Red tattoos have also been associated with phototoxic tattoo reactions.
However, this is thought thought to be due to the addition of cadmium sulfide which enhances and
brightens the red color.
Q/Q(M)-476764 Report a Problem

Which of the following is NOT a form of electrosurgery?
1

Electrocautery
2

Electrodesiccation
3

Electrofulguration
4

Electrocoagulation
5

Electrosection
Q/Q(M)-482909 Report a Problem


Which of the following is NOT a form of electrosurgery?
1

Electrocautery
Electrosurgery is a broad term to describe the use of electricity to produce thermal tissue damage.
Electrocautery is a method of direct heat transfer to tissue by a heated element. No electric current is
transferred.
Q/Q(M)-482909 Report a Problem

Which of the following is the major antioxidant in the human epidermis?
58

1

Superoxide dismutase
2

Coenzyme Q10
3

Ascorbic acid
4

Glutathione
5

Alpha-tocopherol
Q/Q(M)-474757 Report a Problem

Which of the following is the major antioxidant in the human epidermis?
5

Alpha-tocopherol
Research into the preventive role of topical antioxidants in photoaging is based on the free radical theory
of aging. All of the choices above are antioxidants which are naturally found in the skin, however,
alpha-tocopherol is the major antioxidant in human epidermis. Epidermal depletion of this vitamin has
been shown to be an early and sensitive marker of environmental oxidative damage.
Q/Q(M)-474757 Report a Problem


Which of the following cosmetic injectables is FDA-approved for the treatment of lipoatrophy?
1

Radiance
2

Silicone
3

Botox
4

Sculptra
5

Isolagen
Q/Q(M)-474984 Report a Problem

Which of the following cosmetic injectables is FDA-approved for the treatment of lipoatrophy?
4

Sculptra
Sculptra (called New-Fill outside of the US) is a biodegradable filler composed of poly-L-lactic acid, the
same material used in absorbable sutures. It is biocompatible and nonallergenic and was approved by the
FDA in August 2004 for the treatment of HIV-associated lipoatrophy.
Q/Q(M)-474984 Report a Problem


Which of the following local anesthetics has the longest duration of action?
1

Lidocaine
59

2

Bupivacaine
3

Procaine
4

Benzocaine
5

Mepivacaine
Q/Q(M)-476538 Report a Problem

Which of the following local anesthetics has the longest duration of action?
2

Bupivacaine
Bupivacaine is an amide type of local anesthetic. Its duration of action is approximately 3-5 hours and is
the longest one listed. Following bupivacaine is mepivacaine, lidocaine, and procaine. The duration of
action is based on the amine portion of the molecule.
Q/Q(M)-476538 Report a Problem

Treatment with which modality would be most effective for the patient shown?
1

Pulsed dye laser
2

Intense pulsed light
3

Photodynamic therapy
4

Radiofrequency
5

Erbium:YAG laser
Q/Q(M)-475486 Report a Problem



Treatment with which modality would be most effective for the patient shown?
2

Intense pulsed light
The patient shown has multiple solar lentigines. Targeting the melanosomes in these benign, superficial
pigmented lesions would be most successfully accomplished with the intense pulsed light source.
Q/Q(M)-475486 Report a Problem



A patient has a 2.0 cm surgical defect on the left lower eyelid after having Mohs surgery. Which
management option would be the least appropriate?
1

Primary closure
60

2

Secondary intention
3

Full-thickness skin graft
4

Tripier flap
5

Rhombic flap
Q/Q(M)-475364 Report a Problem


A patient has a 2.0 cm surgical defect on the left lower eyelid after having Mohs surgery. Which
management option would be the least appropriate?
2

Secondary intention
In lower eyelid reconstruction, tension should be oriented parallel to the lower eyelid to prevent the
development of ectropion. Secondary intention would be the least appropriate management option in this
location because normal wound contraction would increase the risk of ectropion.
Q/Q(M)-475364 Report a Problem

Each of the following are branches of the internal carotid artery except:
1

Ophthalmic artery
2

Supratrochlear artery
3

Dorsal nasal artery
4

Angular artery
5

Supraorbital artery
Q/Q(M)-474727 Report a Problem
Each of the following are branches of the internal carotid artery except:
4

Angular artery
The angular artery is a branch of the external carotid artery. It arises from the facial artery and courses
superiorly along the nasofacial angle until it reaches the area of the medial canthal tendon. At this
location, the angular artery anastamoses with the dorsal nasal branch of the ophthalmic artery
establishing a communication between the internal and external carotid arterial systems.
Q/Q(M)-474727 Report a Problem

Which of the following surgical prepatory solutions is teratogenic?
1

Povidine-iodine
2

Chlorhexidine
3

Ethyl and Isopropyl alcohol
4

Hexachlorophene
5

Benzalkonium chloride
Q/Q(M)-477431 Report a Problem
Which of the following surgical prepatory solutions is teratogenic?
4

Hexachlorophene
61

Hexachlorophene or pHisoHex has shown to be toxic to developing embryos in animal studies and thus
is prohibited for use in pregnant females. Chlorhexidine is a common ingredient in oral rinses(Peridex)
and surgical prep solutions(Hibiclens) can cause ototoxicity if in contact with the middle ear and ocular
toxicity if in contact with the eyes. Ethyl alcohol is effective against both gram positive and gram
negative organisms, but only when the skin is dry. Both Povidine-Iodine (Betadine)and Benzalkonium
chloride (Zephiran) can cause an allergic contact dermatitis.
Q/Q(M)-477431 Report a Problem

Preliminary studies with phosphatidylcholine demonstrate efficacy in the treatment of:
1

Superficial rhytids
2

Periorbital fat pads
3

Soft tissue augmentation
4

Postinflammatory hyperpigmentation
5

Acne scarring
Q/Q(M)-475344 Report a Problem
Preliminary studies with phosphatidylcholine demonstrate efficacy in the treatment of:
2

Periorbital fat pads
Phosphatidylcholine is a lecithin-derived phospholipid which induces lipolysis when injected into
adipose tissue. Dissolution of fat is likely due to a detergent effect produced by the phosphatidylcholine.
Q/Q(M)-475344 Report a Problem

The most important function of a postoperative wound dressing is:
1

Provide a barrier to infection
2

Decrease tension on the wound
3

Create a moist environment
4

Prevent suture dehiscence
5

Immobilize the wound
Q/Q(M)-474732 Report a Problem

The most important function of a postoperative wound dressing is:
5

Immobilize the wound
Providing hemostasis to a wound is the most important function of a wound dressing. This is particularly
critical in the first few hours postoperatively when the vasoconstrictive effects of epinephrine are
wearing off and the wound is at risk of bleeding or hematoma formation. The provision of a barrier to
infection and creation of a moist, occlusive environment are also necessary functions of a dressing but
are not as critical as immobilizing the wound and ensuring hemostasis.
Q/Q(M)-474732 Report a Problem

Signs and symptoms of lidocaine toxicity include all of the following except:
1

Circumoral numbness
62

2

Ototoxicity
3

Slurred speech
4

Nystagmus
5

Seizure
Q/Q(M)-477415 Report a Problem


Signs and symptoms of lidocaine toxicity include all of the following except:
2

Ototoxicity
The first signs of lidocaine toxicity are CNS symtpoms that resemble inebriation with alcohol. These
symptmoms include stupor, dysarthria, circumoral numbenss and dizziness. Further increases in toxicity
leads to nausea, metallic taste, twitching, and seizures. Ototoxicity is not one of the toxic manifestations
seen with lidocaine toxicity. Without epinephrine the maximum dose of lidocaine 4mg/kg. For a 70 kg
individual, this is 300 mg or 30 ml of a 1% lidocaine solution. For a preparation of lidocaine with
epinephrine the maximum dose is 7mg/kg. For a 70 kg individual, this is 500 mg or 50 ml of a 1%
solution.
Q/Q(M)-477415 Report a Problem


The snap test is performed prior to surgery at which anatomic location?
1

Hand
2

Ear
3

Mouth
4

Eyelid
5

Neck
Q/Q(M)-475355 Report a Problem

The snap test is performed prior to surgery at which anatomic location?
4

Eyelid
The snap test determines the laxity of the lower eyelid. It should be performed prior to surgery, laser
resurfacing or Botox injections of the lower eyelid to assess the potential for ectropion formation. Ratner
D et al. Cutaneous laser resurfacing. J Am Acad Dermatol 1999; 41(3):365-389.
Q/Q(M)-475355 Report a Problem

A patient undergoing Mohs surgery undergoes a rotation flap from the lateral canthus/cheek to close an
infraorbital defect. What is the name of this flap?
1

Rieger flap
2

Mustarde flap
3

O to Z
4

Bilobed
5

Abbe flap
63

Q/Q(M)-482390 Report a Problem


A patient undergoing Mohs surgery undergoes a rotation flap from the lateral canthus/cheek to close an
infraorbital defect. What is the name of this flap?
2

Mustarde flap
This patient had a Mustarde flap, a rotation flap from the lateral canthus or cheek that close the lower
eyelid or infraorbital defect. A Rieger flap is a dorsal nasal rotation flap with backcut at the glabella; it is
also knows as a hatchet flap or glabellar turn-down flap. O to Z is a double rotation flap. A bilobed is a
transposition flap most often used on the nose. Finally, and Abbe flap, or a lip-switch flap, is a lower lip
flat of mucosa, muscle, skin, and subucutis that is transposed superiorly to the repair an upper lip defect
based on inferior labial artery blood supply.
Q/Q(M)-482390 Report a Problem

True statements regarding skin cancer in organ transplant recipients include all of the following except:
1

65 fold increase in development of SCC compared with general population
2

Mohs micrographic surgery indicated for in-transit metastases
3

Cutaneous malignancies develop 3-5 years after organ transplantation
4

Extent of tumor development related to degree of immunosuppression
5

Skin cancer is the most common cancer in transplant patients
Q/Q(M)-474758 Report a Problem

True statements regarding skin cancer in organ transplant recipients include all of the following except:
2

Mohs micrographic surgery indicated for in-transit metastases
In-transit metastases is a common manifestation of metastatic disease.
Q/Q(M)-474758 Report a Problem


Which of the following absorbable sutures lasts the longest?
1

Catgut
2

Polyglycolic acid
3

Polyglactin 910
4

Polydioxanone
5

Polypropylene
Q/Q(M)-474731 Report a Problem


Which of the following absorbable sutures lasts the longest?
4

Polydioxanone
Polydioxanone (PDS) is an absorbable monofilament which lasts approximately 180 days. Catgut has a
variable rate of absorption but typically lasts about 7-14 days. Polyglactin 910 (Vicryl) and polyglycolic
acid (Dexon) are both absorbed in about 90 days. Polypropylene (Prolene) is not an absorbable suture.
64

Q/Q(M)-474731 Report a Problem

In a patient who is allergic to paraphenylinediamine, which of the following anesthetics should be
avoided?
1

Bupivicaine
2

Etidocaine
3

Mepivicaine
4

Prilocaine
5

Benzocaine
Q/Q(M)-477393 Report a Problem


In a patient who is allergic to paraphenylinediamine, which of the following anesthetics should be
avoided?
5

Benzocaine
Paraphenyelenediamine is a common allergen found in permanent hair dyes. In individuals who are
allergic to paraphenylendiamine, may also have allergic reactions to ester anesthetics, like benzocaine.
The other answer choices are amide anesthetics. Other crossreactants to paraphenyelediamine include
the preservative parabens, sulfonamides, and thiazide diuretics.
Q/Q(M)-477393 Report a Problem


The post-operative complication shown in the photograph is most commonly seen with which type of
reconstruction?
1

Rhombic flap
2

Nasolabial flap
3

Rotation flap
4

Full-thickness skin graft
5

Secondary intention
Q/Q(M)-474979 Report a Problem
65



The post-operative complication shown in the photograph is most commonly seen with which type of
reconstruction?
2

Nasolabial flap
Trap door deformity is believed to result from insufficient undermining. This surgical complication is
most often associated with the nasolabial transposition flap. Intralesional corticosteroids may be
beneficial in improving the cosmetic outcome.
Q/Q(M)-474979 Report a Problem

The use of EMLA cream is contraindicated in patients with which of the following?
1

Atopic dermatitis
2

Neomycin allergy
3

Sickle cell anemia
4

Methemoglobinemia
5

Peripheral neuropathy
Q/Q(M)-474739 Report a Problem
The use of EMLA cream is contraindicated in patients with which of the following?
4

Methemoglobinemia
EMLA is a topical anesthetic composed of a eutectic mixture of 2.5% lidocaine and 2.5% prilocaine.
The major concern when using EMLA is the potential risk of methemoglobinemia. Patients with
glucose-6-phosphate deficiency and patients taking methemoglobin-inducing agents (dapsone,
phenytoin, sulfonamides) are more susceptible to developing methemoglobinemia. In addition, care
should be taken when using EMLA in infants less than 3 months of age because of the incomplete
maturation of their NADH-methemoglobinemia reductase system.
Q/Q(M)-474739 Report a Problem

Which type of collagen is the first to be deposited in a healing wound?
1

Type I collagen
66

2

Type II collagen
3

Type III collagen
4

Type IV collagen
5

Type VII collagen
Q/Q(M)-477370 Report a Problem

Which type of collagen is the first to be deposited in a healing wound?
3

Type III collagen
Coagulation/Inflammation, tissue formation, and scar/remodeling are the three phases important in
wound healing. Collagen deposition occurs during the second phase. Fibroblasts migrate into the wound
along the fibrin-fibronectin matrix deposited in the initial clot. The fibroblasts subsequently produce
type I and III collagen, elastin, and proteoglycans. Collagen type III is the predominant collagen in early
wound healing.
Q/Q(M)-477370 Report a Problem

The type of repair depicted in the photograph is a:
1

Rhombic flap
2

Nasolabial flap
3

Rotation flap
4

Full-thickness skin graft
5

Island pedicle flap
Q/Q(M)-474976 Report a Problem



The type of repair depicted in the photograph is a:
67

5

Island pedicle flap
The island pedicle flap is a type of advancement flap commonly used for medial cheek defects. The
closure can often be camouflaged in the melolabial fold.
Q/Q(M)-474976 Report a Problem


A patient has a 1.5cm surgical defect on the right nasal sidewall that reveals the lateral cartilage. Which
management option would be the least appropriate?
1

Primary closure
2

Secondary intention
3

Full-thickness skin graft
4

Cheek transposition flap
5

Forehead flap
Q/Q(M)-474752 Report a Problem
A patient has a 1.5cm surgical defect on the right nasal sidewall that reveals the lateral cartilage. Which
management option would be the least appropriate?
3

Full-thickness skin graft
Full-thickness skin grafts (FTSGs) depend upon a viable, vascular bed at the recipient site in order to
survive. Exposed cartilage is an avascular tissue and therefore would not be able to support the
metabolic requirements of a FTSG.
Q/Q(M)-474752 Report a Problem
A patient with a squamous cell carcinoma of the helical rim had excisional surgery followed by closure
of the wound with an advancement flap. Which would be the most appropriate antibiotic for this patient
to receive postoperatively?
1

Dicloxacillin
2

Cephalexin
3

Erythromycin
4

Ciprofloxacin
5

Trimethoprim-sulfamethoxazole
Q/Q(M)-474736 Report a Problem

A patient with a squamous cell carcinoma of the helical rim had excisional surgery followed by closure
of the wound with an advancement flap. Which would be the most appropriate antibiotic for this patient
to receive postoperatively?
4

Ciprofloxacin
Infection after cutaneous surgery is typically due to Staphylococcal and Streptococcal organisms.
However, Pseudomonas aeruginosa may be normal flora of the ear and thus can potentially complicate
ear surgery by causing severe infection of the external ear canal (malignant otitis externa). Ciprofloxacin
would be the antibiotic of choice for this patient because it is effective against Pseudomonas species.
Q/Q(M)-474736 Report a Problem
68


On average, how much does a full-thickness graft contract when removed from a donor site?
1

15%
2

30%
3

50%
4

60%
5

80%
Q/Q(M)-478768 Report a Problem
On average, how much does a full-thickness graft contract when removed from a donor site?
1

15%
On the average, how much does a full-thickness graft may contract by about 15% when removed from a
donor site.
Q/Q(M)-478768 Report a Problem

A Mohs surgery fellow experiences burning and tingling of her fingertips 6 months after starting her
fellowship. She most likely has been exposed to which one of the following chemicals?
1

Chlorhexidine gluconate
2

Hematoxylin
3

Povodine-iodine
4

Hexachlorophene
5

Toluidine blue
Q/Q(M)-474716 Report a Problem

A Mohs surgery fellow experiences burning and tingling of her fingertips 6 months after starting her
fellowship. She most likely has been exposed to which one of the following chemicals?
4

Hexachlorophene
Hexachlorophene is an antibacterial agent that was first introduced in 1944. Its use was discontinued in
the 1970s when it was discovered to have neurotoxic side effects.
Q/Q(M)-474716 Report a Problem

Which cosmetic injectable provides dermal augmentation through the harvesting of a patients
fibroblasts?
1

Restylane
2

Isolagen
3

Hylaform
4

Juvaderm
5

Dermalogen
69

Q/Q(M)-475348 Report a Problem

Which cosmetic injectable provides dermal augmentation through the harvesting of a patients
fibroblasts?
2

Isolagen
Isolagen is an emerging technology whereby a patients own fibroblasts are isolated from a skin
biopsy, reproduced and then re-introduced into the patients treatment site.
Q/Q(M)-475348 Report a Problem
Epinephrine should be avoided with anesthetics in which of the following conditions?
1

Hypothyroidism
2

Diabetes Mellitus
3

Renal failure
4

Liver failure
5

Pheochromocytoma
Q/Q(M)-477389 Report a Problem
Epinephrine should be avoided with anesthetics in which of the following conditions?
5

Pheochromocytoma
Epinephrine should be avoided in patients with a pheochromocytoma.
Q/Q(M)-477389 Report a Problem


Which of the following sutures is best used for mucous membranes?
1

Catgut
2

Silk
3

Polyglactin 9, 10
4

Nylon
5

Polydioxanone
Q/Q(M)-477406 Report a Problem


Which of the following sutures is best used for mucous membranes?
2

Silk
Silk is a natural nonabsorbable suture used mainly for eyelid, lip, or intraoral surgeries as they are more
comfortable to the patient on these surfaces. This suture has excellent handling as it is less stiff and
flows easily through soft tissue. However, it has high tissue reactivity, low tensile strength, and is
associated with a higher risk of infection.
Q/Q(M)-477406 Report a Problem



The following flaps are types of transposition flaps except:
70

1

Rhombic flap
2

Nasolabial
3

Bilobed
4

A to T
5

Z plasty
Q/Q(M)-478037 Report a Problem

The following flaps are types of transposition flaps except:
4

A to T
The Z plasty, Bilobed, Rhombic, nasolabial flaps are all examples of transposition flaps. In each of
these, the flap is transposed over a normal piece of skin to be placed in the recipient site. Whereas, the A
to T flap, is an advancement flap, in which tissue is moved in a linear direction to cover the defect.
Q/Q(M)-478037 Report a Problem


Reticulate eythema is a side effect seen with which treatment?
1

Mesotherapy
2

Sclerotherapy
3

Diode laser
4

Cryotherapy
5

Pulsed dye laser
Q/Q(M)-475374 Report a Problem

Reticulate eythema is a side effect seen with which treatment?
3

Diode laser
Reticulate erythema is a recently-reported side effect of diode laser treatment. High energy fluences and
a history of chilblains are believed to be predisposing risk factors. Laser treatment should be
discontinued at the first sign of this complication.
Q/Q(M)-475374 Report a Problem


To avoid injury to the facial nerve, undermining in the region of the temple should be performed at
which anatomic level?
1

Dermal-epidermal junction
2

Reticular dermis
3

Subcutaneous fat
4

Above superficial fascia (SMAS)
5

Above the frontalis muscle
Q/Q(M)-474720 Report a Problem
71


To avoid injury to the facial nerve, undermining in the region of the temple should be performed at
which anatomic level?
3

Subcutaneous fat
In the region of the temple, lateral to the eyebrow, the temporal branch of the facial nerve lies in the
superficial fascia. Once above the eyebrow, the nerve is located deep to the frontalis muscle. In the
region of the temple, it is therefore prudent to undermine in the superficial subcutaneous fat.
Q/Q(M)-474720 Report a Problem


A Z-plasty is performed to:
1

Decrease tension on a wound
2

Obtain wound eversion
3

Shorten the length of an excision
4

Correct dog-ears
5

Make use of excess tissue
Q/Q(M)-474747 Report a Problem

A Z-plasty is performed to:
1

Decrease tension on a wound
A Z-plasty is a transposition flap used most commonly in the treatment of contractures and scars. The
main advantages to performing a Z-plasty are to decrease the tension on a wound, change the orientation
of a scar, and camouflage a scar by breaking it up into smaller components.
Q/Q(M)-474747 Report a Problem

The surgical instrument shown in the photograph is a:
1

Needle holder
2

Hemostat
3

Gradle scissor
4

Towel clamp
5

Iris scissor
Q/Q(M)-475482 Report a Problem


The surgical instrument shown in the photograph is a:
72

2

Hemostat
A hemostat is shown in the photograph. This instrument is commonly used in cutaneous surgery for
clamping off blood vessels.
Q/Q(M)-475482 Report a Problem


A patient with a basal cell carcinoma on the lower eyelid has Mohs surgery. Once tumor-free margins
were obtained, the surgeon repairs the wound with a full thickness skin graft. Which of the following
statements is true about this repair method?
1

Graft should be at least 25% larger than the defect
2

Graft rarely survives in this location
3

Graft should be the same size as the defect
4

Healing by secondary intention would minimize the risk of ectropion
5

Graft should be at least 25% smaller than the defect
Q/Q(M)-475353 Report a Problem

A patient with a basal cell carcinoma on the lower eyelid has Mohs surgery. Once tumor-free margins
were obtained, the surgeon repairs the wound with a full thickness skin graft. Which of the following
statements is true about this repair method?
1

Graft should be at least 25% larger than the defect
To account for wound contraction and minimize the risk of ectropion, full thickness skin grafts on the
lower eyelid should be sized such that the graft is at least 25% larger than the actual size of the wound.
Jewett BS and Shockley WW. Reconstructive options for periocular defects. Otolaryngol Clin North Am
2001; 34(3):601-625.
Q/Q(M)-475353 Report a Problem


The trapdoor effect that can occur with a flap can be avoided by which of the following techniques:
1

Wide undermining
2

Using additional sutures
3

Performing the flap in a two staged procedure
4

Using a flap with length to width ratio of less than 3:1
5

Avoid thinning the flap
Q/Q(M)-477391 Report a Problem

The trapdoor effect that can occur with a flap can be avoided by which of the following techniques:
1

Wide undermining
Wide undermining of the primary defect can release tension and prevent tenting or the trap door
deformity of the flap.
Q/Q(M)-477391 Report a Problem


73

All of the following are alpha-hydroxy acids except:
1

Lactic acid
2

Citric acid
3

Glycolic acid
4

Tartaric acid
5

Salicylic acid
Q/Q(M)-477376 Report a Problem

All of the following are alpha-hydroxy acids except:
5

Salicylic acid
Alpha-hydroxy acids are naturally occurring carboxylic acids found in many foods. The alpha-hydroxy
acids include glycolic, lactic, malic, citric, and tartaric acids. Factors that determine the intensity of the
peel include the concentration of the acid, pH, degree of buffering, vehicle formulation, frequency of
application, conditions of delivery, and the length of time the acid is placed on the skin. Salicylic acid is
a type of beta-hydroxy acid.
Q/Q(M)-477376 Report a Problem


The pain associated with Botulinum A Exotoxin injection is attributed to the:
1

Needle gauge
2

PH
3

Preservative-free saline
4

Exotoxin
5

Temperature
Q/Q(M)-477123 Report a Problem

The pain associated with Botulinum A Exotoxin injection is attributed to the:
3

Preservative-free saline
Botulinum A exotoxin is used for multiple reasons, most often for the treatment of dynamic facial lines.
In a double-blind, randomized controlled study, investigators found that botulinum A exotoxin
reconstituted with preservative-containing saline less painful than with preservative-free saline.
Q/Q(M)-477123 Report a Problem


Which structual component of local anesthetics (e.g. lidocaine) is responsible for the onset of activity?
1

Aromatic ring
2

Intermediate chain
3

Amine end
4

Length of the carbon chain
5

None of the above
Q/Q(M)-476545 Report a Problem
74


Which structual component of local anesthetics (e.g. lidocaine) is responsible for the onset of activity?
1

Aromatic ring
Local anesthetics like lidocaine contain 3 principle structural elements. The aromatic ring determines the
onset of activity, the intermediate chain defines the class (amide vs. ester), and the amine end is
responsible for the duration of action.
Q/Q(M)-476545 Report a Problem


Which is the following is true regarding cellulite?
1

It is caused by lipoatrophy of fat.
2

Adipocytes in the gluteofemoral region are more responsive to lipolysis.
3

Norepinephrine is the only hormone that affects lipolysis.
4

It occurs in up to 50% of postpubertal females.
5

Caucasian women tend to get cellulite more than Asian women.
Q/Q(M)-482514 Report a Problem

Which is the following is true regarding cellulite?
5

Caucasian women tend to get cellulite more than Asian women.
According to the JAAD CME article in March 2010, cellulite is caused by the herniation of
subcutaneous fat, not lipoatrophy, within fibrous connective tissue. It is prevalent in almost all
postpubertal females. It is most notable in the pelvic region, lower limbs, and abdomen. The adipocytes
in the gluteofemoral region are larger and are influenced by female sex hormones. They are
metabolically more stable and resistant to lipolysis. The hormones that acutely affect lipolysis in
adipocytes are catecholamines (epinephrine and norepinephrine) and insulin.
Q/Q(M)-482514 Report a Problem

Which cosmetic filler substance is contraindicated in the glabella?
1

Bovine collagen
2

Human collagen
3

Hyaluronic acid
4

Calcium hydroxyapatite
5

Poly-L-lactic acid
Q/Q(M)-482682 Report a Problem

Which cosmetic filler substance is contraindicated in the glabella?
1

Bovine collagen
As placement of bovine collagen is typically deeper in the tissues compared to other filler, its use has
resulted in vascular occlusion and necrosis in the glabellar area, and is therefore contraindicated.
Q/Q(M)-482682 Report a Problem
75



Which of the following is a potential side effect of ambulatory tumescent liposuction?
1

Muscle atrophy
2

Breast enlargement
3

Decreased appetite
4

Abdominal perforation
5

Pulmonary embolus
Q/Q(M)-474744 Report a Problem
Which of the following is a potential side effect of ambulatory tumescent liposuction?
2

Breast enlargement
Breast enlargement is a relatively common and unexpected side effect of tumescent liposuction. Since
the majority of these patients report increased breast size in the absence of weight gain, some authors
postulate shifting hormone ratios as the etiology of this paradoxical breast augmentation. Abdominal
perforation, respiratory failure and pulmonary embolus are complications that are seen almost
exclusively in liposuction patients that receive general anesthesia or intravenous sedation.
Q/Q(M)-474744 Report a Problem

All of the following statements are true regarding nerves blocks on the face except:
1

Anesthesia is placed around a nerve trunk
2

Lidocaine with epinephrine may be used
3

Numbness occurs in areas other than the operative site
4

Tissue distortion is minimized
5

Hemostasis is achieved using epinephrine
Q/Q(M)-475372 Report a Problem

All of the following statements are true regarding nerves blocks on the face except:
5

Hemostasis is achieved using epinephrine
While regional nerve blocks offer many advantages in cutaneous surgery, hemostasis is not one of them.
Since the anesthetic is injected distant to the operative site, the vasoconstrictive effects of the
epinephrine, and hence hemostasis, are not provided at the surgical site.
Q/Q(M)-475372 Report a Problem


For high risk surgical patients, when is prophylaxis for infective endocarditis ideally first given?
1

2 days prior to the procedure
2

1 day prior to the procedure
3

1 week prior to the procedure
4

60 minutes prior to the procedure
5

after the procedure
76

Q/Q(M)-482914 Report a Problem

For high risk surgical patients, when is prophylaxis for infective endocarditis ideally first given?
4

60 minutes prior to the procedure
Antibiotics are usually given 60 minutes prior to the procedure. The American Heart Association
advises it can be given up to 2 hours after the procedure if not given before. Infections are usually
polymicrobial, but staph and Strep are of prime concern.
Q/Q(M)-482914 Report a Problem


The antiptosis subdermal suspension threads used in facial rejuvenation are composed of:
1

Poly-L-lactic acid
2

Polyglactin
3

Polypropylene
4

Polydioxanone
5

Polyglycolic acid
Q/Q(M)-474980 Report a Problem

The antiptosis subdermal suspension threads used in facial rejuvenation are composed of:
3

Polypropylene
The Aptos subdermal suspension thread technique is a minimally invasive procedure which targets the
ptotic changes seen with facial aging. 2-0 and 3-0 polypropylene threads are tunneled in the dermis to
lift and suspend the skin and subcutaneous tissue.
Q/Q(M)-474980 Report a Problem


A patient with a large nasal tip defect had reconstruction of his wound with a forehead flap. After what
postoperative interval should the patient schedule the inset of this flap?
1

1 week
2

3 weeks
3

2 months
4

3 months
5

6 months
Q/Q(M)-474740 Report a Problem

A patient with a large nasal tip defect had reconstruction of his wound with a forehead flap. After what
postoperative interval should the patient schedule the inset of this flap?
2

3 weeks
The forehead flap is often the reconstructive option of choice for large nasal defects. The flap has a
vertically-oriented blood supply which is supplied primarily from the supratrochlear arteries. Pedicle
division and inset of this flap are typically performed 3 weeks after the initial surgery.
77

Q/Q(M)-474740 Report a Problem

Which one of the substances is not a component of a Jessner's peel?
1

Ethanol
2

Salicylic acid
3

Lactic acid
4

Phenol
5

Resorcinol
Q/Q(M)-482297 Report a Problem

Which one of the substances is not a component of a Jessner's peel?
4

Phenol
Jessner's is a light-medium chemical peel that contains salicylic acid, lactic acid, ethanol, and resorcinol.
Phenol is used in deep peels, ie the Baker-Gordon peel which also contains croton oil, water, and
septisol. Phenol may induce cardiac arrythmias and liver or renal failure; hydration is important in all
phase of treatment.
Q/Q(M)-482297 Report a Problem


Aging skin demonstrates all of the following characteristic changes except:
1

Loss of elasticity
2

Hypertrophy of sebaceous glands
3

Decreased number of Langerhans cells
4

Increase in subcutaneous fat
5

Decreased dermal collagen
Q/Q(M)-474756 Report a Problem


Aging skin demonstrates all of the following characteristic changes except:
4

Increase in subcutaneous fat
The loss and redistribution of subcutaneous fat is a characteristic finding of the aging face. The
forehead, temporal fossae, malar cheeks and perioral region are the most commonly affected areas.
Knowledge of senescent changes in fat distribution has altered the cosmetic surgeons approach of the
aging face to one which includes augmentation rather than one of simply lifting and tightening the skin.
Q/Q(M)-474756 Report a Problem

Você também pode gostar